Sunteți pe pagina 1din 55

Lógica y Razonamiento matemático

UTECO
Maestro Julio A. Manzueta

UNIVERSIDAD TECNOLÓGICA DEL CIBAO ORIENTAL


COTUÍ, REPUBLICA DOMINICANA ABRIL DE 2018
Contenido
Unidad I: Lógica. .......................................................................................................................... 4
Ejercicios 1.1 .............................................................................................................................. 6
Lógica clásica ......................................................................................................................... 7
Juicio ...................................................................................................................................... 7
Inferencia ............................................................................................................................... 7
Verdad .................................................................................................................................... 8
Lógica binaria........................................................................................................................ 8
Lógica ternaria ...................................................................................................................... 8
Lógica difusa.......................................................................................................................... 8
Lógica proposicional ............................................................................................................. 8
Proposición ............................................................................................................................ 8
Ejercicios 1.2 .............................................................................................................................. 8
Razonamiento ...................................................................................................................... 10
Argumento ........................................................................................................................... 10
Ejercicios 1.3 ............................................................................................................................ 12
Forma argumental .............................................................................................................. 12
Validez de un argumento .................................................................................................... 12
Leyes lógicas ........................................................................................................................ 12
Ejercicios 1.4 ............................................................................................................................ 13
Unidad II: Teoría De La Demostración Matemática. .............................................................. 15
Principio de inducción Matemática. ...................................................................................... 15
Teorema 2.1 .......................................................................................................................... 16
Lema 2.1 ............................................................................................................................... 16
Lema 2.2 ............................................................................................................................... 16
Teorema 2.2 .......................................................................................................................... 17
Teorema 2.4 .......................................................................................................................... 17
Teorema 2.5 .......................................................................................................................... 17
Teorema 2.6 .......................................................................................................................... 17
Lema 2.3 ............................................................................................................................... 18
Lema 2.4 ............................................................................................................................... 18
Ejercicios 2.1 ............................................................................................................................ 18
La inducción “a saltos”....................................................................................................... 18
Proposición 2.1 .................................................................................................................... 19

1
Proposición 2.2 .................................................................................................................... 20
Demostración Vacía (prueba por vacuidad) ......................................................................... 21
Proposición 2.3 .................................................................................................................... 21
Demostración ....................................................................................................................... 21
Demostración Trivial .............................................................................................................. 21
Proposición 2.4 .................................................................................................................... 21
Demostración directa .............................................................................................................. 21
Proposición 2.5 .................................................................................................................... 21
Demostración indirecta .......................................................................................................... 22
Demostración por contradicción ....................................................................................... 23
Demostración por contrarrecíproca .................................................................................. 24
Demostración por contraejemplos ..................................................................................... 24
Proposición 2.9 .................................................................................................................... 24
Teorema 2.9 (Teorema fundamental de la aritmética) ......................................................... 25
Ejercicios 2.2 ............................................................................................................................ 25
Unidad III: Esquemas Conceptuales. ........................................................................................ 26
Mapas conceptuales y aplicaciones.................................................................................... 26
Ejercicios 3.1 ............................................................................................................................ 30
Diagramas de flujos. ........................................................................................................... 31
Ejercicios 3.2 ............................................................................................................................ 32
Unidad IV: Resolución De Problemas. ...................................................................................... 33
Técnicas generales ................................................................................................................... 34
Pensamiento lateral ............................................................................................................. 34
Principio de discontinuidad ............................................................................................... 34
Imitación .............................................................................................................................. 35
Factores emocionales .......................................................................................................... 35
Bloqueos mentales ............................................................................................................... 35
i. La metodología de Pólya ................................................................................................. 36
ii. El aporte de Alan Schoenfeld .......................................................................................... 38
iii. MODELO DE MASOM-BURTON-STACEY ........................................................... 40
iv. MODELO DE MIGUEL DE GUZMÁN .................................................................... 40
v. Método de singapur ......................................................................................................... 46
Ejercicios 4.1 ............................................................................................................................ 48
VII. Bibliografía Y Referencias .................................................................................................. 53

2
Lógica y Razonamiento matemático

Descripción de la asignatura:
Este curso trata de dotar a los participantes de diferentes herramientas que lo capacitan para un
mejor razonamiento matemático. Iniciamos con la lógica simbólica: conectivos lógicos,
proposiciones, tablas de verdad y aplicación de las mismas al razonamiento. Incluye una gama de
métodos de demostración matemático, así como estrategias para abordarlos. Otro bloque se refiere
a esquemas y redes de razonamiento, los cuáles ayudan a organizar las informaciones, leyes y
conceptos, entre las que están: diagramas de flujos, mapas conceptuales y mapas mentales.
Comprende también la Resolución de Problemas, como estrategias para desarrollar el
razonamiento inductivo y la implementación de diversas estrategias para resolver problemas.

Metodología
Este curso será una combinación de exposiciones del profesor y del concurso interactivo
participante-facilitador; mediante el trabajo cooperativo dentro del aula. Este curso precisa de
discusiones de prácticas y trabajos de investigación sobre temas del programa u otros afines que
permitan el logro de los objetivos.

Objetivo General
Proveer oportunidades de aprendizajes para que los futuros maestros se apropien del lenguaje
matemático y puedan usarlo con propiedad, puedan adquirir la habilidad para la sistematización
del conocimiento matemático a través de diferentes recursos como los esquemas y redes
conceptuales, puedan dominar la teoría de la demostración Matemática mediante diferentes
métodos y resolver problemas de forma adecuada; aplicando los pasos necesarios y aplicación de
diferentes estrategias de solución. Además de, proporcionar al participante herramientas que le
permitan reconocer, elaborar y determinar la validez de razonamientos lógicos tanto deductivos
como inductivos.

Al finalizar esta asignatura cada participante mostrara las siguientes Competencias:


➢ Domina las principales operaciones lógicas, simples y compuestas.
➢ Exhiben dominios en la construcción de diagramas de flujos, redes y mapas conceptuales.
➢ Aplican diversos métodos de demostración matemática con precisión y conciencia.
➢ Distinguen un problema de un ejercicio, dominan los pasos de resolución de problemas y
pueden resolverlo aplicando diversas estrategias de solución.
➢ Identifica y aplica las diferentes leyes de la lógica en procesos de argumentación, al
llevarlas al lenguaje natural.
➢ Desarrolla competencias para la construcción de funciones lógicas en programas de
computación, como las hojas de cálculo o de lenguajes de programación.

3
Unidad I: Lógica.

Tales de Mileto (639 a.c), quien es considerado como el primer representante de la filosofía
occidental: tanto, así como para reconocérsele como el iniciador de la indagación racional sobre
el universo, a Tales de Mileto se atribuye plantear explicaciones de la naturaleza sin hacer
referencia a lo sobrenatural. Es así, como los precursores de la filosofía, llamados los
«presocráticos», representaron una innovación en el pensamiento, al tratar de explicar las cosas
por sí mismas.

En el período Socrático, los filósofos pasaron de preocuparse por los temas de la naturaleza a
ocuparse en el hombre. En este período aparecen los sofistas, quienes profundizaron en el “arte de
discutir”, a ellos debemos lo que en la lógica se denomina un sofisma, argumentos que parecen
válidos pero que realmente no lo son.

Originalmente logos significa palabra o discurso, por lo que en un principio se definió la lógica
como la rama de la gramática que se ocupaba de ciertas formas de lenguaje.

Como la palabra es la expresión, o manifestación del pensamiento y el pensamiento racional es la


base de la filosofía, puede decirse en general, que la lógica es la ciencia del pensamiento racional;
es importante aclarar que la lógica no se ocupa del contenido de los pensamientos sino de la manera
o forma de los pensamientos.

En respuesta a la necesidad de construir argumentos, para defender o refutar pensamientos de los


demás, Aristóteles, considerado por los griegos. “El padre de la lógica”, creo métodos sistemáticos
para analizar y evaluar dichos argumentos, para lo cual desarrolló la lógica proposicional
estableciendo procedimientos para determinar la verdad o falsedad de proposiciones compuestas.
El gran matemático Gottfried Leibniz fue el primero en intentar reformar la lógica clásica,
planteando que la dependencia lógica entre proposiciones es demostrada reduciendo argumentos
complejos en simples.

En 1847 el matemático inglés George Boole junto con Augustus de Morgan hizo notar el
parentesco entre las operaciones lógicas con las matemáticas, pues a partir de los operadores
aritméticos de adición, multiplicación y sustracción crearon los operadores lógicos equivalentes
de unión, intersección y negación; además formularon los principios del razonamiento simbólico
y el análisis lógico. A Boole se le atribuye la invención de las tablas de verdad para comprobar la
veracidad de proposiciones compuestas. Este trabajo fue retomado por Bertrand Russell y Alfred
Whitehead en 1910 en su obra “Principio Matemático”, quienes codificaron la lógica simbólica en
su presente forma. *

*: Georffrey Acevedo González. Lógica Matemática. Medellín, 2011

4
La lógica se puede clasificar como lógica tradicional (lógica no formal) y lógica simbólica (lógica
formal). En la lógica no formal se considera la destreza para interpretar y distinguir un
razonamiento correcto de un razonamiento incorrecto como un producto de la experiencia humana
obtenida en la relación con el mundo circundante. Mientras que la lógica formal, se encarga de
investigar, desarrollar y establecer los principios fundamentales que siguen la validez de la
inferencia. En el pensamiento simbólico, las palabras se manipulan, según las reglas establecidas,
como si fueran simples signos sin preocuparse por su sentido.

La lógica ofrece métodos que enseñan cómo formar proposiciones, evaluar sus valores de verdad
y determinar si unas conclusiones se pueden deducir correctamente a partir de proposiciones
supuestas; además, la lógica es una ciencia que se interesa por las relaciones existentes entre las
proposiciones, con el fin de obtener precisión, claridad y generalidad en los razonamientos.
Logrando la precisión mediante el uso de símbolos, los cuales tienen como función primordial
eliminar las ambigüedades que la estructura del lenguaje ordinario no puede evitar con facilidad.

Los lenguajes naturales y formales tienen puntos en común, en principio, se tiene la existencia de
un conjunto finito llamado alfabeto, el cual está constituido de símbolos simples llamados
comúnmente letras.

El lógico, está interesado esencialmente en la corrección del proceso completo de razonamiento.


El lógico pregunta: ¿Tiene solución el problema?, ¿se sigue la conclusión de las premisas que se
han afirmado?, ¿las premisas proporcionan buenas razones para aceptar la conclusión?

Si el problema queda resuelto, si las premisas proporcionan las bases adecuadas para afirmar la
conclusión, si afirmar las premisas constituye una verdadera garantía para afirmar la verdad de la
conclusión, entonces el razonamiento es correcto. De lo contrario, es incorrecto. Esta distinción
entre el razonamiento correcto e incorrecto es el problema central con el que trata la lógica.

Algunas definiciones de lógica se presentan a continuación para que hagas un cuidadoso análisis.

1. La lógica es el estudio de los métodos y principios que se usan para distinguir el razonamiento
correcto del incorrecto.
2. La lógica es la ciencia que estudia las leyes del pensamiento.
3. La lógica como la ciencia del razonamiento simbólico.

5
Ejercicios 1.1

1. Discutir las definiciones de lógica antes presentadas, sustentar alguna e intentar refutar otra.
______________________________________________________________________________
______________________________________________________________________________
______________________________________________________________________________
______________________________________________________________________________
______________________________________________________________________________
______________________________________________________________________________
______________________________________________________________________________
______________________________________________________________________________
2. ¿Qué entiendes por lógica?
______________________________________________________________________________
______________________________________________________________________________
______________________________________________________________________________
______________________________________________________________________________
______________________________________________________________________________
______________________________________________________________________________
3. ¿Podríamos hacer un debate de ideas sin hacer uso de la lógica?
______________________________________________________________________________
______________________________________________________________________________
______________________________________________________________________________
______________________________________________________________________________
______________________________________________________________________________
______________________________________________________________________________

4. ¿Qué recuerdas de la evolución histórica de la lógica?


______________________________________________________________________________
______________________________________________________________________________
______________________________________________________________________________
______________________________________________________________________________
______________________________________________________________________________
________________________________________________________________________

5. Porqué es importante la competencia lógico matemática para apropiar nuevo conocimiento.


______________________________________________________________________________
______________________________________________________________________________
______________________________________________________________________________
______________________________________________________________________________
______________________________________________________________________________
______________________________________________________________________________

6
Lógica clásica

La lógica clásica estaría constituida por un conjunto de cálculos lógicos equivalentes al cálculo
presentado por Bertrand Russell y Alfred N. Whitehead en sus Principia Mathematica (1910-
1913). Equivalentemente, la lógica clásica es aquella que se desarrolla desde Aristóteles hasta las
aportaciones de Alfred Tarski, hacia mediados de los años 30 del siglo XX.

Juicio

El juicio es una conexión enunciativa de conceptos. Ahora bien, en esta conexión o enlace vamos
a encontrar los elementos fundamentales que constituyen su estructura o forma como éstos se
presentan. Los juicios pueden ser clasificados del siguiente modo:

a) Juicios universales afirmativos, en estos juicios el predicado se extiende a toda una clase de
objetos o seres, en términos afirmativos.

Ejemplo: Todos los maestros de Cotuí son Excelentes.

b) Juicios universales negativos, en estos juicios el predicado niega que determinados atributos
pertenezcan a toda una clase de objetos, o sea, que el predicado no conviene a todos los casos del
sujeto.

Ejemplo: Ningún profesor recibe todo lo que merece.

c) Juicios particulares afirmativos. Señala que ciertas características o atributos pertenecen a


algunos miembros de una clase de objetos o seres.

Ejemplo: Algunos profesores tienen dominio de los contenidos que imparten.

d) Juicios particulares negativos. Indican que ciertas características o atributos no pertenecen a


algunos miembros de una clase de objetos o seres.

Ejemplo: Algunos maestros no tienen dominio de teoría de números.

Inferencia

es el proceso por el cual se llega a una proposición y se afirma sobre la base de una o más
proposiciones aceptadas como punto inicial del proceso. Para determinar si una inferencia es
correcta, el lógico examina las proposiciones que constituyen los puntos inicial y final de este
proceso, así como las relaciones que existen entre ellos.

7
Verdad

Una verdad lógica o verdad matemática es una fórmula bien formada de un lenguaje formal que
es verdadera bajo todas las interpretaciones de los componentes, las verdades lógicas se conocen
como fórmulas lógicamente válidas (que tienen validez lógica).

Lógica binaria

La lógica binaria es la que trabaja con variables binarias y operaciones lógicas del Álgebra de
Boole. Así, las variables sólo toman dos valores discretos.

Lógica ternaria

Se llama lógica ternaria o lógica trivalente a cualquier sistema lógico multivaluado en el que hay
tres valores de verdad, indicando Verdadero, Falso y algún otro valor indeterminado. Esto contrasta
con las más comunes lógicas bivalentes (tales como la clásica lógica proposicional o la lógica
booleana), que contemplan únicamente Verdadero o Falso.

Lógica difusa

Lógica Difusa es una lógica multivaluada que permite representar matemáticamente la


incertidumbre y la vaguedad, proporcionando herramientas formales para su tratamiento

Lógica proposicional

La lógica proposicional es una lógica binaria en la cual el elemento básico es la proposición y los
valores de verdad que admiten las proposiciones son sólo dos, verdadero y falso, 0 y 1 u otro que
se acuerde.

Proposición

Una proposición lógica es un enunciado lingüístico que debe cumplir con la condición de ser
susceptible de poder ser verdadero o falso.

Ejercicios 1.2

I. Escribe los conectivos y sus valores de verdad

II. Escribe tres proposiciones simples, nómbralas como se acostumbra en lógica


proposicional, a partir de estas, construye 10 proposiciones compuestas y la forma
proposicional correspondiente a cada una.

III. Escribe un ensayo sobre tablas de verdad (redacte pensando en que pueda entenderlo
un estudiante de 3ro de secundaria).

8
IV. Escribe un ensayo sobre la importancia y aplicación de las tablas de verdad (incluya
aplicaciones a la electrónica y a las ciencias puras).

V. Realice la tabla de verdad de las siguientes formas proposicionales, clasifíquelas según


el resultado; escoja diez y explique la importancia de estas diez al desarrollo de la
ciencia.
1) ~(~𝑝) ⟺ 𝑝
2) (𝑝⋁𝑞) ⟺ (𝑞⋁𝑝)
3) (𝑝⋀𝑞) ⟺ (𝑞⋀𝑝)
4) (𝑝 ⟺ 𝑞) ⟺ (𝑞 ⟺ 𝑝)
5) [(𝑝⋁𝑞)⋁𝑟] ⟺ [𝑝⋁(𝑞⋁𝑟)]
6) [(𝑝⋀𝑞)⋀𝑟] ⟺ [𝑝⋀(𝑞⋀𝑟)]
7) [𝑝⋁(𝑞⋀𝑟)] ⟺ [(𝑝⋁𝑞)⋀(𝑝⋁𝑟)]
8) [𝑝⋀(𝑞⋁𝑟)] ⟺ [(𝑝⋀𝑞)⋁(𝑝⋀𝑟)]
9) (𝑝⋁𝑞) ⟺ 𝑝
10) (𝑝⋀𝑞) ⟺ 𝑝
11) ∼ (𝑝⋁𝑞) ⟺ (∼ 𝑝 ⋀ ∼ 𝑞)
12) ∼ (𝑝⋀𝑞) ⟺ (∼ 𝑝 ⋁ ∼ 𝑞)
13) (𝑝⋁𝑞) ⟺∼ (∼ 𝑝 ⋀ ∼ 𝑞)
14) (𝑝⋀𝑞) ⟺ (∼ 𝑝 ⋁ ∼ 𝑞)
15) (𝑝 → 𝑞) ⟺ (∼ 𝑝⋁𝑞)
16) (𝑝 → 𝑞) ⟺∼ (𝑝 ⋀ ∼ 𝑞)
17) (𝑝⋁𝑞) ⟺ (∼ 𝑝 → 𝑞)
18) (𝑝⋀𝑞) ⟺∼ (𝑝 →∼ 𝑞)
19) [(𝑝 → 𝑟)⋀(𝑞 → 𝑟] ⟺ [(𝑝⋁𝑞) → 𝑟]
20) [(𝑝 → 𝑞)⋀(𝑝 → 𝑟)] ⟺ [(𝑝 → (𝑞⋀𝑟)]
21) (𝑝 ⟺ 𝑞) ⟺ [(𝑝 → 𝑞)⋀(𝑞 → 𝑝)]
22) [(𝑝⋀𝑞) → 𝑟] ⟺ [𝑝 → (𝑞 → 𝑟)]
23) (𝑝 → 𝑞) ⟺ [(𝑝 ⋀ ∼ 𝑞) → 𝑐]
24) 𝑝 → (𝑝⋁𝑞)
25) (𝑝⋀𝑞) → 𝑝
26) [𝑝⋀(𝑝 → 𝑞)] ⇒ 𝑞
27) [(𝑝 → 𝑞)⋀ ∼ 𝑞)] ⇒∼ 𝑝
28) [(𝑝⋁𝑞)⋀ ∼ 𝑝)] ⇒ 𝑞
29) 𝑝 ⇒ [𝑞 → (𝑝⋀𝑞)]
30) [(𝑝 ⟺ 𝑞)⋀(𝑞 ⟺ 𝑟)] ⇒ (𝑝 ⟺ 𝑟)
31) [(𝑝 → 𝑞)⋀(𝑞 → 𝑟)] ⇒ (𝑝 → 𝑟)
32) (𝑝 → 𝑞) ⇒ [(𝑝⋁𝑟) → (𝑞⋁𝑟)]
33) (𝑝 → 𝑞) ⇒ [(𝑝⋀𝑟) → (𝑞⋀𝑟)]
34) (𝑝 → 𝑞) ⇒ [(𝑞 → 𝑟) → (𝑝 → 𝑟)]
35) (𝑝 → 𝑞)⋀(𝑟 → 𝑠) ⇒ [(𝑝⋁𝑟) → (𝑞⋁𝑠)]
36) (𝑝 → 𝑞)⋀(𝑟 → 𝑠) ⇒ [(𝑝⋀𝑟) → (𝑞⋀𝑠)]
37) (𝑝 → 𝑞)⋀(𝑟 → 𝑠) ⇒ [(∼ 𝑞 ⋁ ∼ 𝑠) → (∼ 𝑝 ⋁ ∼ 𝑟)]
38) (𝑝 → 𝑞)⋀(𝑟 → 𝑠) ⇒ [(∼ 𝑞⋀ ∼ 𝑠) → (∼ 𝑝⋀ ∼ 𝑟)]

9
Razonamiento

Razonamiento es el proceso y el resultado de razonar, es un proceso mental a partir del cual, se


puede partir de una o de varias premisas para arribar a una conclusión.

Argumento

Un argumento es la expresión lingüística de un razonamiento, consta de premisas y conclusión. La


conclusión de un argumento es la proposición que se afirma con base en las otras proposiciones
del argumento, y estas otras proposiciones, que son afirmadas (o supuestas) como apoyo o razones
para aceptar la conclusión, son las premisas de ese argumento.

Un argumento es un grupo de proposiciones de las cuales una, la conclusión, pretende derivarse o


seguirse de las otras, que son las premisas. Las proposiciones son típicamente enunciadas en
oraciones declarativas, pero en ocasiones aparecen como órdenes, preguntas retóricas o frases
nominales.

El tipo más simple de argumento consiste sólo de una premisa y una conclusión, que se dice está
implicada por, o se sigue de, la primera. Un ejemplo en el que cada una de ellas se enuncia en una
oración independiente es el siguiente:

República Dominicana es un país en vía de desarrollo. Por tanto, debe poner énfasis y ser vigilante
en la inversión en educación.

En el ejemplo anterior se enuncia primero la premisa y luego la conclusión. Pero el orden en


el que son enunciadas no es importante desde el punto de vista lógico. Un
argumento en el que la conclusión se enuncia en la primera oración y la
premisa en la segunda es:

Los casos de corrupción, como el de Odebrecht, dificultan la aplicación de la ley. Los casos de
corrupción son notorios cuando afectan los sentimientos y distorsionan la capacidad de apreciación
de los jueces.

En algunos argumentos, la premisa y la conclusión se enuncian en la misma oración. Como en el


siguiente:

Como en Cotuí llueve mucho, los habitantes de Cotuí no salen sin paraguas.

Algunas veces, la conclusión precede a la premisa en un argumento de una sola oración, como en
el siguiente ejemplo:

Promover un estudiante que no sabe perjudica la sociedad del presente y del futuro, ya que los
que al momento saben pueden desanimarse y los que, en el futuro, dependerán de dicho promovido
sufrirán la ineptitud de aquel individuo.

10
Para listar las premisas de un argumento, no podemos apelar simplemente al número de oraciones
en las que están escritas. Debemos notar que "premisa" y "conclusión" son términos relativos:
una y la misma proposición puede ser una premisa en un argumento y una conclusión en otro.

Ninguna proposición por sí misma, considerada en forma aislada, es una premisa ni una
conclusión. Es una premisa solamente cuando aparece como supuesto de un argumento. Es una
conclusión solamente cuando aparece en un argumento y pretende fundamentarse en otras
proposiciones del argumento.

La conclusión de un argumento puede hallarse en medio de diferentes premisas que


se ofrecen en su apoyo.

Las proposiciones se pueden afirmar en forma de preguntas retóricas, que se usan para hacer
afirmaciones más bien que para plantear preguntas, aun cuando se expresan en forma interrogativa.

A veces, la naturaleza proposicional de un elemento constitutivo de un argumento se oculta bajo


su expresión como frase nominal en lugar de como oración declarativa.

Hay frases que ayudan a identificar una premisa tales como:

❖ Como es indicado por


❖ La razón es que
❖ Por las siguientes razones
❖ Se puede inferir de
❖ Se puede derivar de
❖ Se puede deducir de
❖ En vista de que

Hay frases que ayudan a identificar la conclusión tales como:

➢ Puesto que
➢ Dado que
➢ A causa de
➢ Porque
➢ Pues
➢ Se sigue de
➢ Como muestra

11
Ejercicios 1.3

Identifique las premisas y la conclusión en cada argumento.

1. Muestra de razonamiento obtuso es introducir el tema del "libre ejercicio" de la religión en


el presente caso. Nadie está forzado a ir a las clases de religión y ninguna institución
religiosa va a llevar su culto y su credo a los salones de clase de las escuelas públicas. Un
estudiante no está obligado a tomar instrucción religiosa. En sus propios deseos están la
forma o el momento de expresar su devoción religiosa, si la hay.

2. Si el derecho penal prohíbe el suicidio, esto no es un argumento válido para la Iglesia; y,


además, la prohibición es ridícula, pues ¿qué pena puede atemorizar a una persona que no
tiene miedo ni siquiera a la muerte?

3. Si una persona dice, amo a Dios y odio a mi hermano, está mintiendo: porque si no ama a
su hermano, a quien ha visto, ¿cómo puede amar a Dios, a quien no ha visto?

Forma argumental

El uso del lenguaje complica nuestro problema. Ciertas características accidentales o engañosas
de las formulaciones en el lenguaje pueden hacer más difícil la labor de investigar las relaciones
lógicas entre las proposiciones. Es razón suficiente para que al estudiar un argumento se requiera
tenerlo en lenguaje lógico. A la expresión simbólica de un argumento se le llama forma argumental
.

Validez de un argumento

La verdad y la falsedad se refieren a proposiciones, no a argumentos. Y los atributos de validez e


invalidez pueden pertenecer solamente a los argumentos deductivos, nunca a las proposiciones.
Hay una conexión entre la validez o invalidez de un argumento y la verdad o falsedad de sus
premisas y de su conclusión; un argumento puede ser válido aun cuando una o más de sus premisas
no sean verdaderas.

➢ Si un argumento es válido y su conclusión es falsa, no todas sus premisas pueden ser


verdaderas.
➢ Si un argumento es válido y sus premisas son verdaderas, con toda certeza la conclusión
debe ser también verdadera.
➢ Algunos argumentos perfectamente válidos tienen conclusiones falsas, pero tal género de
argumentos debe al menos tener alguna premisa falsa.

Leyes lógicas

Una ley lógica es una función proposicional correspondiente a un argumento que es válido para
cualquier valor de verdad de las proposiciones que la forman.

12
Ejercicios 1.4
A continuación, presentamos un conjunto de formas proposicionales (𝑓𝑝) verifique que
corresponden a argumentos válidos y por tanto son leyes lógicas.

1. Doble negación
~(~𝑝) ⟺ 𝑝
2. Leyes conmutativas
a) (𝑝⋁𝑞) ⟺ (𝑞⋁𝑝)
b) (𝑝⋀𝑞) ⟺ (𝑞⋀𝑝)
c) (𝑝 ⟺ 𝑞) ⟺ (𝑞 ⟺ 𝑝)

3. Leyes asociativas

a) [(𝑝⋁𝑞)⋁𝑟] ⟺ [𝑝⋁(𝑞⋁𝑟)]
b) [(𝑝⋀𝑞)⋀𝑟] ⟺ [𝑝⋀(𝑞⋀𝑟)]

4. Leyes distributivas
a) [𝑝⋁(𝑞⋀𝑟)] ⟺ [(𝑝⋁𝑞)⋀(𝑝⋁𝑟)]
b) [𝑝⋀(𝑞⋁𝑟)] ⟺ [(𝑝⋀𝑞)⋁(𝑝⋀𝑟)]

5. Leyes de la impotencia
a) (𝑝⋁𝑞) ⟺ 𝑝
b) (𝑝⋀𝑞) ⟺ 𝑝

6. Leyes de De Morgan
a) ∼ (𝑝⋁𝑞) ⟺ (∼ 𝑝⋀ ∼ 𝑞)
b) ∼ (𝑝⋀𝑞) ⟺ (∼ 𝑝⋁ ∼ 𝑞)
c) (𝑝⋁𝑞) ⟺∼ (∼ 𝑝⋀ ∼ 𝑞)
d) (𝑝⋀𝑞) ⟺ (∼ 𝑝⋁ ∼ 𝑞)

7. Leyes de Implicación
a) (𝑝 → 𝑞) ⟺ (∼ 𝑝⋁𝑞)
b) (𝑝 → 𝑞) ⟺∼ (𝑝 ⋀ ∼ 𝑞)

8. Ley de Equivalencia
(𝑝 ⟺ 𝑞) ⟺ [(𝑝 → 𝑞)⋀(𝑞 → 𝑝)]

9. Ley de exportación
[(𝑝⋀𝑞) → 𝑟] ⟺ [𝑝 → (𝑞 → 𝑟)]

10. Reducción al absurdo


(𝑝 → 𝑞) ⟺ [(𝑝⋀ ∼ 𝑞) → 𝑐]

11. Ley de adición


𝑝 → (𝑝⋁𝑞)

13
12. Ley de simplificación
(𝑝⋀𝑞) → 𝑝

13. Modus ponens


[𝑝⋀(𝑝 → 𝑞)] ⇒ 𝑞

14. Modus tollens


[(𝑝 → 𝑞)⋀ ∼ 𝑞)] ⇒∼ 𝑝

15. Silogismo disyuntivo


[(𝑝⋁𝑞)⋀ ∼ 𝑝)] ⇒ 𝑞

16. Transitividad de la doble implicación


[(𝑝 ⟺ 𝑞)⋀(𝑞 ⟺ 𝑟)] ⇒ (𝑝 ⟺ 𝑟)

17. Transitividad de la implicación o silogismo hipotético


[(𝑝 → 𝑞)⋀(𝑞 → 𝑟)] ⇒ (𝑝 → 𝑟)

18. Dilemas constructivos


a) (𝑝 → 𝑞)⋀(𝑟 → 𝑠) ⇒ [(𝑝⋁𝑟) → (𝑞⋁𝑠)]
b) (𝑝 → 𝑞)⋀(𝑟 → 𝑠) ⇒ [(𝑝⋀𝑟) → (𝑞⋀𝑠)]

19. Dilemas destructivos

a) (𝑝 → 𝑞)⋀(𝑟 → 𝑠) ⇒ [(∼ 𝑞⋁ ∼ 𝑠) → (∼ 𝑝⋁ ∼ 𝑟)]


b) (𝑝 → 𝑞)⋀(𝑟 → 𝑠) ⇒ [(∼ 𝑞⋀ ∼ 𝑠) → (∼ 𝑝⋀ ∼ 𝑟)]

14
Unidad II: Teoría De La Demostración Matemática.

Definición.
Una demostración matemática es un argumento deductivo para una afirmación matemática.
Es un conjunto de juicios debidamente enlazados que permiten concluir una afirmación
matemática (Teorema, lema, etc.) a partir de unas afirmaciones que se consideran ciertas
(hipótesis, axiomas, teoremas, etc.).

Las demostraciones se clasifican según la estructura del razonamiento. Aquí presentamos sólo lo
necesario para que el lector pueda realizar una demostración.

El conjunto de los números naturales se puede caracterizar mediante los siguientes axiomas,
introducidos por el matemático italiano Giuseppe Peano en 1899:
1. Hay un elemento especial 0 ∈ ℕ.
2. Para todo 𝑛 ∈ ℕ existe un ´único elemento 𝑛+ ∈ ℕ llamado el sucesor de 𝑛.
3. Para todo 𝑛 ∈ ℕ, 𝑛+ ≠ 0
4. Si 𝑛, 𝑚 ∈ ℕ, 𝑛+ = 𝑚+ 𝑒𝑛𝑡𝑜𝑛𝑐𝑒𝑠 𝑛 = 𝑚 .
5. Si 𝐴 es un subconjunto de ℕ tal que: 0 ∈ 𝐴 y 𝑛+ ∈ 𝐴 siempre que 𝑛 ∈ 𝐴 entonces 𝐴 = ℕ.

En la formulación de los axiomas de Peano se supone de antemano la existencia del conjunto ℕ.


El 3 establece la existencia de un primer número natural que es 0. El axioma 4 indica que números
naturales diferentes tienen sucesores diferentes. El axioma 5 se conoce como El Principio de
Inducción Matemática, PIM.

Principio de inducción Matemática.

Definición.
Las siguientes ecuaciones definen la adición en ℕ. Para todo 𝑚, 𝑛 ∈ ℕ:
𝑚 + 0 = 𝑚,
𝑚 + 𝑛+ = (𝑚 + 𝑛)+ .

Como todo número natural distinto de cero es el sucesor de un número natural la adición resulta
bien definida.

15
Teorema 2.1
La adición de números naturales es asociativa, es decir: Para todo 𝑛, 𝑚, 𝑘 ∈ ℕ
(𝑛 + 𝑚) + 𝑘 = 𝑛 + (𝑚 + 𝑘).
Demostración (por inducción)

Sea 𝐴 = {𝑘 ∈ ℕ | (𝑛 + 𝑚) + 𝑘 = 𝑛 + (𝑚 + 𝑘) para todo 𝑛, 𝑚 ∈ ℕ }.


i. Por definición de adición, (𝑛 + 𝑚) + 0 = 𝑛 + 𝑚 = 𝑛 + (𝑚 + 0) ⇒ 0 ∈ 𝐴
ii. Supongamos que 𝐾 ∈ 𝐴, es decir que para todo 𝑛, 𝑚 ∈ ℕ, (𝑛 + 𝑚) + 𝑘 = 𝑛 + (𝑚 + 𝑘)
Entonces,
(𝑛 + 𝑚) + 𝑘 + = [(𝑛 + 𝑚) + 𝑘]+ (definición de suma)
= [𝑛 + (𝑚 + 𝑘)]+ (hip. inducción)
+
= 𝑛 + (𝑚 + 𝑘) (definición de suma)
+
= 𝑛 + (𝑚 + 𝑘 ) (definición de suma)
⇒ 𝑘+ ∈ 𝐴
∴𝐴=ℕ ∎

Lema 2.1
Para todo 𝑚 ∈ ℕ, 0 + 𝑚 = 𝑚.

Demostración. Sea 𝐴 = {𝑚 ∈ ℕ | 0 + 𝑚 = 𝑚}.

i. 0 + 0 = 0 ⇒ 0 ∈ 𝐴,
ii. Supongamos que 𝑚 ∈ 𝐴, es decir, que 0 + 𝑚 = 𝑚.
Entonces:
0 + 𝑚+ = (0 + 𝑚)+ (definición suma)
= 𝑚+ (hip. inducción)
+
⇒𝑚 ∈𝐴
∴𝐴=ℕ ∎

Lema 2.2
Para todo 𝑚, 𝑛 ∈ ℕ, 𝑚+ + 𝑛 = (𝑚 + 𝑛)+ .
Demostración.
Sea 𝐴 = {𝑛 ∈ ℕ |𝑚+ + 𝑛 = (𝑚 + 𝑛)+ 𝑝𝑎𝑟𝑎 𝑡𝑜𝑑𝑜 𝑚 ∈ ℕ}.
i. 𝑝𝑎𝑟𝑎 𝑡𝑜𝑑𝑜 𝑚 ∈ ℕ, 𝑚+ + 0 = 𝑚+ = (𝑚 + 0)+ ⇒ 0 ∈ 𝐴
ii. Supongamos que 𝑛 ∈ 𝐴, es decir, que para todo 𝑚 ∈ ℕ, 𝑚+ + 𝑛 = (𝑚 + 𝑛)+ .
Entonces
para todo 𝑚 ∈ ℕ, 𝑚+ + 𝑛+ = (𝑚+ + 𝑛)+
= [(𝑚 + 𝑛)+ ]+ (hip. inducción)
+ +
= (𝑚 + 𝑛 ) (definición suma)
⇒ 𝑛+ ∈ 𝐴
∴𝐴=ℕ ∎

16
Teorema 2.2
La adición de números naturales es conmutativa, es decir: Para todo 𝑛, 𝑚, ∈ ℕ
𝑛 + 𝑚= 𝑛 + 𝑚
Demostración (por inducción)

Sea 𝐴 = {𝑛 ∈ ℕ | 𝑚 + 𝑛 = 𝑛 + 𝑚 para todo 𝑚 ∈ ℕ }.


i. 𝑚 + 0 = 𝑚 = 0 + 𝑚 ⇒ 0 ∈ 𝐴,
ii. Supongamos que 𝑛 ∈ 𝐴, es decir 𝑛 + 𝑚 = 𝑛 + 𝑚
Entonces,
para todo 𝑚 ∈ ℕ, 𝑚 + 𝑛+ = (𝑚 + 𝑛)+ (definición suma)
= (n + m)+ (hip. inducción)
= 𝑛+ + 𝑚 (Lema 2.2).
+
⇒𝑛 ∈𝐴
∴𝐴=ℕ ∎

Teorema 2.3
Si 𝑛, 𝑚 y 𝑘 son números naturales tales que 𝑚 + 𝑘 = 𝑛 + 𝑘, entonces 𝑚 = 𝑛.

Definición.
Las siguientes ecuaciones definen la multiplicación en ℕ. Para todo 𝑚, 𝑛 ∈ ℕ:
𝑚0 = 0,
𝑚𝑛+ = 𝑚𝑛 + 𝑚.

Como todo número natural distinto de cero es el sucesor de un número natural la adición resulta
bien definida.

Teorema 2.4
La multiplicación es distributiva con respecto a la adición, es decir: para todo 𝑚, 𝑛, 𝑘 ∈ ℕ,
𝑚(𝑛 + 𝑘) = 𝑚𝑛 + 𝑚𝑘.

Teorema 2.5

La multiplicación de números naturales es asociativa: para todo 𝑛, 𝑚, 𝑘 ∈ ℕ,


(𝑚𝑛)𝑘 = 𝑚(𝑛𝑘).

Teorema 2.6

La multiplicación de números naturales es conmutativa. Es decir: Para todo 𝑚, 𝑛 ∈ ℕ,


𝑚𝑛 = 𝑛𝑚.

17
Lema 2.3
Para todo 𝑚 ∈ ℕ, tenemos 0𝑚 = 0.

Lema 2.4
Para todo 𝑚, 𝑛 ∈ ℕ, tenemos 𝑚+ 𝑛 = 𝑚𝑛 + 𝑛.

Ejercicios 2.1
1. Demostrar el teorema 2.3
2. Demostrar el teorema 2.4
3. Demostrar el teorema 2.5
4. Demostrar el teorema 2.6
5. Demostrar el lema 2.3
6. Demostrar el lema 2.4
7. Demostrar que todo número natural diferente de cero es de la forma 𝑛+ para algún 𝑛 ∈ ℕ.
8. Demostrar que para todo 𝑛 ∈ ℕ, 𝑛+ = 𝑛 + 0+ .
9. Si m y n son números naturales tales que 𝑚 + 𝑛 = 0, probar que 𝑚 = 0 y 𝑛 = 0.
10. Demostrar que si 𝑚, 𝑛 ∈ ℕ entonces 𝑚 + 𝑛 ∈ ℕ y 𝑚𝑛 ∈ ℕ.
11. Probar que si 𝑛, 𝑚 ∈ ℕ son tales que 𝑚𝑛 = 0 entonces 𝑚 = 0, o 𝑛 = 0.

La inducción “a saltos”
Puede darse el caso de que sea muy sencillo demostrar que, si el resultado que nos piden se cumple
para 𝑛, entonces se cumple para 𝑛 + 3, pero muy difícil demostrar que, si se cumple para 𝑛,
entonces se cumple para 𝑛 + 1.
¿Qué hacemos entonces?
Bueno, si se cumple para 𝑛 = 1, entonces sabemos que se cumple para 𝑛 = 4, luego, también para
𝑛 = 7, y para 10,13,16, . .. Con esto no nos basta, pero, si además demostramos que el resultado
se cumple para 𝑛 = 2, habremos demostrado también que se cumple para 𝑛 = 5,8,11, . . ., y si
finalmente demostramos el resultado para 𝑛 = 3, lo habremos demostrado para
𝑛 = 6,9,12, . .. ¡Ahora sí lo habremos demostrado para todo entero positivo!

Nótese entonces, que no necesitamos aplicar la inducción “de uno en uno”, sino que, podemos
ir dando saltos, con tal de que los casos iniciales que demostremos sean los suficientes para
que, “dando saltos” de algún tamaño hayamos demostrado que la propiedad se sigue
cumpliendo, así, podemos llegar hasta cualquier entero positivo, desde alguno de los casos
iniciales.

18
Proposición 2.1
Demostrar que todo cuadrado perfecto o da resto cero, o da resto 1, al dividir por 4.

Demostración
Los cuadrados perfectos son todos los números de la forma 𝑘 2 , para 𝑘 = 1,2,3, . . ., y es fácil
ver que,
(𝑘 + 2)2 = 𝑘 2 + 4(𝑘 + 1)
entonces
𝑘 2 y (𝑘 + 2)2 dan el mismo resto al dividir por 4
Entonces, si el resultado es cierto para 𝑘, es cierto para 𝑘 + 2.
Necesitamos (y nos basta con) demostrar que el resultado es cierto para los casos iniciales
𝑘 = 1, para el que 𝑘 2 = 1 da resto 1 al dividir por 4, y para 𝑘 = 2, para el que 𝑘 2 = 4 da
resto 0 al dividir por 4.
Claramente podemos llegar a cualquier entero positivo par dando saltos de 2 en 2, desde
𝑘 = 2, y para cualquier entero positivo impar dando saltos de 2 en 2, desde 𝑘 = 1, así que,
está probado. ∎

En los casos anteriores, nos han pedido que demostráramos algo. ¿Qué tal si nos piden hallar
una expresión y demostrarla, pero no sabemos cómo es la expresión?
Supongamos que nos piden hallar cuánto vale la suma de los 𝑛 primeros cuadrados perfectos.
Por comodidad, llamemos 𝑆𝑛 a esta suma.
Para 𝑛 = 1, … … … . . … … …. 𝑆1 = 12 = 1
Para 𝑛 = 2, … … … . … … … . . . 𝑆2 = 12 + 22 = 5
Para 𝑛 = 3, … … … … … … … . 𝑆3 = 12 + 22 + 32 = 14
Pero, ¿cuál es la fórmula general?
Como la suma de 1,2,3, . . . , 𝑛 se puede expresar, como un polinomio de grado 2 en 𝑛,
podemos pensar que la suma de 12 + 22 + ⋯ + 𝑛2 tal vez se pueda expresar como un
polinomio de grado 3 en 𝑛 (aumentando en cada caso en 1 el grado del polinomio que nos da
el resultado respecto al grado que tienen los sumandos). En el caso más general, parece
modelarse 12 + 22 + 32 … + 𝑛2 = 𝑎𝑛3 + 𝑏𝑛2 + 𝑐𝑛 + 𝑑 ?
En esta expresión, 𝑎, 𝑏, 𝑐 𝑦 𝑑 son constantes que necesitamos calcular. Para hallar estas
constantes, utilizamos los casos más sencillos de calcular, es decir, aquellos para los que 𝑛
vale 1, 2, 3 y 4. Así, si la suma de los 𝑛 primeros cuadrados perfectos tuviera efectivamente
la forma anterior, podemos generar un sistema de 4 ecuaciones con 4 incógnitas, que
resolvemos por eliminación de variables, restando a cada ecuación la anterior del sistema:
𝑎+𝑏+𝑐+𝑑 =1
{ 8𝑎 + 4𝑏 + 2𝑐 + 𝑑 = 5 }
27𝑎 + 9𝑏 + 3𝑐 + 𝑑 = 14
64𝑎 + 16𝑏 + 4𝑐 + 𝑑 = 30

19
𝑎+𝑏+𝑐+𝑑 =1
𝑖𝑚𝑝𝑙𝑖𝑐𝑎 𝑞𝑢𝑒, { 7𝑎 + 3𝑏 + 𝑐 = 4 }
19𝑎 + 5𝑏 + 𝑐 = 9
37𝑎 + 7𝑏 + 𝑐 = 16
𝑎+𝑏+𝑐+𝑑 =1
𝑙𝑢𝑒𝑔𝑜, { 7𝑎 + 3𝑏 + 𝑐 = 4 }
12𝑎 + 2𝑏 = 5
18𝑎 + 2𝑏 = 7
𝑎+𝑏+𝑐+𝑑 =1
𝑑𝑒 𝑑𝑜𝑛𝑑𝑒 , { 7𝑎 + 3𝑏 + 𝑐 = 4 }
12𝑎 + 2𝑏 = 5
6𝑎 = 2

Hallamos entonces de la cuarta ecuación el valor de 𝑎, sustituimos en la tercera para hallar 𝑏,


en la segunda para hallar 𝑐 y finalmente hallamos 𝑑 de la primera ecuación, resultando en la
siguiente hipótesis:

Proposición 2.2
2𝑛3 +3𝑛2 +𝑛
12 + 22 + 32 + ⋯ + 𝑛2 = 6
Esto, si es cierto (todavía seguimos sin saberlo), es lo que queremos demostrar y lo haremos
por inducción.
Claramente, sustituyendo 𝑛 = 1, tenemos que el miembro de la derecha vale en efecto 1, así
que se cumple para el caso inicial.
Supongamos que se cumple para 𝑛, es decir
2 2 2 2
2𝑛3 + 3𝑛2 + 𝑛
1 +2 +3 +⋯+𝑛 =
6
Sumando (𝑛 + 1)2 en ambos miembros de la igualdad, se tiene:
2𝑛3 +3𝑛2 +𝑛
12 + 22 + 32 + ⋯ + 𝑛2 + (𝑛 + 1)2 = + 𝑛2 + 2𝑛 + 1
6
2𝑛3 +9𝑛2 +13𝑛+6
= 6
2(𝑛3 +3𝑛2 +3𝑛+1)+3(𝑛2 +2𝑛+1)+(𝑛+1)
= 6
2(𝑛+1)3 +3(𝑛+1)2 +(𝑛+1)
= 6
+
Con lo que también se cumpliría para 𝑛 = 𝑛 + 1, y ahora sí sabemos que el resultado que
nosotros mismos hemos conjeturado es cierto, y hemos acabado.

Es posible que, después de conjeturar una hipótesis, es decir, de pensar que algo que se nos
pide hallar toma una determinada forma, nos demos cuenta de que no es así, porque
encontremos algún caso en el que no se cumple; por ejemplo, en el caso anterior, hemos
hallado una hipótesis que vale para 𝑛 = 1,2,3,4, pero a lo mejor podría haber fallado para
𝑛 = 5. Tendría entonces que volver atrás y conjeturar otra hipótesis distinta.

20
Demostración Vacía (prueba por vacuidad)
Una demostración de este tipo se construye estableciendo que el valor verdadero de la
hipótesis 𝑝 es falso. Una demostración de la forma 𝐹 ⇒ 𝑞, para construirla lo único que hay
que establecer es el antecedente falso.

Proposición 2.3
Todos los números primos pares mayores que 2 son cuadrados perfectos.

Demostración
Como todo número par mayor que 2 es divisible por 2, se tiene que cualquier número par
mayor que 2 no es primo, luego, no existe número primo par mayor que 2, así, por vacuidad
se tiene que todos los números primos pares mayores que 2 son cuadrados perfectos.

Demostración Trivial
Se construye una demostración de este tipo, probando que el valor verdadero de la conclusión
es verdad. Una demostración de la forma 𝑝 ⇒ 𝑉. Para dar una prueba trivial lo único que hay que
establecer es que el consecuente es verdadero.

Proposición 2.4
Demostrar que si un número entero es de la forma 3𝑘 + 1 entonces, el cuadrado del número
es mayor o igual que cero.

Demostración
𝑎2 = |𝑎| 2 = |𝑎| ∗ |𝑎| ≥ 0 ∀𝑎 ∈ ℝ
Por tanto, es trivial que si un número entero es de la forma 3𝑘 + 1 entonces, el cuadrado del
número es mayor o igual que cero.

Demostración directa
Una demostración de este tipo muestra que la verdad de la conclusión 𝑞 , se sigue lógicamente
de la verdad de la hipótesis 𝑝. La demostración empieza asumiendo que 𝑝 es verdad para
después, utilizando cualquier información disponible, así como teoremas probados con
anterioridad, probar que 𝑞 es verdad.

Proposición 2.5
Probar que

Si 𝑎 y 𝑏 son enteros con 𝑏 ≥ 1, entonces existe algún entero 𝑚 tal que

𝑚𝑏 ≤ 𝑎 < (𝑚 + 1)𝑏

21
Demostración

Sea 𝑘 un número entero, sea U = {d: d = a − kb ≥ 0}

Entonces, hay dos casos posibles:

i. Si 𝑎 ≥ 0 , entonces, 𝑎 − 0𝑏 ≥ 0 , entonces 𝑎 ∈ 𝑈
ii. Si 𝑎 < 0, entonces, 𝑎 − 𝑎𝑏 = 𝑎(1 − 𝑏) ≥ 0, entonces, 𝑎 − 𝑎𝑏 ∈ 𝑈.

De i y ii se sigue que 𝑈 es un conjunto no vacío, así que, por el principio del buen orden, 𝑈
tiene un elemento mínimo, sea este 𝑑 = 𝑎 − 𝑚𝑏 ≥ 0, entonces, 𝑎 − (𝑚 + 1)𝑏 < 0, luego,
𝑚𝑏 ≤ 𝑎 < (𝑚 + 1)𝑏 ∎

Teorema 2.7
La relación ≤ define un orden sobre ℕ.
Demostración
i. Con 0 ∈ ℕ y 𝑚 = 𝑚 + 0 se tiene que Para todo 𝑚 ∈ ℕ, 𝑚 ≤ 𝑚, así la relación ≤
es reflexiva.
ii. Si 𝑛, 𝑚 son números naturales tales que 𝑚 ≤ 𝑛 y 𝑛 ≤ 𝑚, entonces existen 𝑝, 𝑞 ∈ ℕ
tales que 𝑛 = 𝑚 + 𝑝 y 𝑚 = 𝑛 + 𝑞. Luego, 𝑚 = (𝑚 + 𝑝) + 𝑞 = 𝑚 + (𝑝 +
𝑞). Por lo tanto, 𝑝 + 𝑞 = 0 y, en consecuencia, 𝑝 = 𝑞 = 0, lo que implica 𝑚 =
𝑛. Así que la relación ≤ es antisimétrica.
iii. Si 𝑚, 𝑛, 𝑟 ∈ ℕ son tales que 𝑚 ≤ 𝑛 y 𝑛 ≤ 𝑟, entonces 𝑛 = 𝑚 + 𝑝 y 𝑟 = 𝑛 + 𝑞
donde 𝑝, 𝑞 ∈ ℕ, y por lo tanto 𝑟 = (𝑚 + 𝑝) + 𝑞 = 𝑚 + (𝑝 + 𝑞) donde 𝑝 + 𝑞 ∈
ℕ, luego 𝑚 ≤ 𝑟. Así que la relación ≤ es transitiva.
Por i, ii, iii se concluye que la relación ≤ define un orden. ∎

Demostración indirecta
En el método de demostración indirecta, debes empezar preguntándote: “¿Cómo, o cuándo, debo
concluir que la tesis es verdadera?” Esta pregunta debes hacerla de forma general. Esta pregunta,
la llamaremos en lo que sigue la pregunta clave. Una pregunta clave bien planteada no debería
contener ni símbolos ni otras notaciones (salvo números) del problema que se está considerando.
La llave para muchas demostraciones es formular correctamente la tal pregunta clave. Una vez
que has planteado la pregunta clave, tu paso siguiente en este método será responderla. Una de las
22
dificultades que pueden surgir en el método de demostración indirecta es la posibilidad de más de
una pregunta clave en algún paso. Elegir la correcta tiene más de arte que de ciencia. En algunos
casos, habrá solamente una pregunta clave obvia; en otros casos, deberás proceder por ensayo y
error. Aquí es donde tu intuición, esfuerzo, creatividad, tus diagramas, etc., pueden jugar un papel
importante.

Una norma general, es dejar que la información que encierran las hipótesis te ayude a elegir
la tal pregunta. Al margen de la pregunta clave que finalmente plantees, el siguiente paso será
responderla, primero en general y luego aplicada a la situación en cuestión.

Demostración por contradicción


La demostración de un teorema 𝑝 → 𝑞 diremos que es por contradicción cuando suponiendo
que la conclusión, 𝑞 es falsa y utilizando la hipótesis, 𝑝 y otros teoremas y equivalencias
lógicas establecidas previamente, se llega a una contradicción.

Definición.
Dados 𝑚, 𝑛 ∈ ℕ decimos que:𝑚 ≤ 𝑛 si existe 𝑝 ∈ ℕ tal que 𝑛 = 𝑚 + 𝑝.
Como es usual, definimos m < n si 𝑚 ≤ 𝑛 y 𝑚 ≠ 𝑛.

Podemos observar como consecuencia de la definición que 𝑚 < 𝑛 si y solo si 𝑛 = 𝑚 + 𝑝+


para algún 𝑝 ∈ ℕ.

Teorema 2.8 (Ley de la tricotomía).


Dados 𝑚, 𝑛 ∈ ℕ una y sólo una de las siguientes afirmaciones es verdadera,
𝑚 < 𝑛
𝑚 = 𝑛
𝑛 < 𝑚

Lema 2.5
Si 𝑚, 𝑛 ∈ ℕ todas las afirmaciones siguientes son falsas:
1) 𝑚 < 𝑛 𝑦 𝑚 = 𝑛.
2) 𝑛 < 𝑚 𝑦 𝑛 = 𝑚.
3) 𝑚 < 𝑛 𝑦 𝑛 < 𝑚.

23
Demostración.
Supongamos que 𝑚 < 𝑛 y 𝑚 = 𝑛, entonces 𝑛 = 𝑚 + 𝑝+ , donde 𝑝 𝑛 ∈ ℕ y 𝑚 = 𝑛, de
donde 𝑝+ = 0. Esto contradice el axioma 3 de Peano. Luego es falsa la proposición supuesta,
la afirmación 1.
Análogamente demostramos que la afirmación 2 es falsa.
Supongamos que 𝑚 < 𝑛 y 𝑛 < 𝑚 entonces 𝑛 = 𝑚 + 𝑝+ y 𝑚 = 𝑛 + 𝑞 + de donde
𝑛 = (𝑛 + 𝑞 + ) + 𝑝+ = 𝑛 + (𝑞 + + 𝑝+ ) = 𝑛 + (𝑞 + + 𝑝)+ de donde (𝑞 + + 𝑝)+ = 0,
lo que contradice el axioma 3 de Peano, luego, la afirmación 3 es falsa.

Proposición 2.8
Demostrar que, si el cuadrado de un número entero es impar, entonces el número es impar.

Demostración
Sea 𝑛 un número entero impar, supongamos que 𝑛2 es impar, entonces, por la definición de
número impar y la de número par, existen dos números enteros 𝑎 y 𝑏 tales que
𝑛2 = 2𝑎 + 1 y 𝑛 = 2𝑏 luego,
𝑛2 = 2𝑎 + 1 ∧ 𝑛2 = 4𝑏 2 por lo tanto, 2𝑎 + 1 = 4𝑏 2 de donde, 1 = 4𝑏 2 − 2𝑎 =
2(2𝑏 2 − 𝑎). Pero, si 𝑎 y 𝑏 son enteros, entonces, 2𝑏 2 − 𝑎 también lo es (lo llamaremos
𝑚), tendremos que hemos encontrado un número entero 𝑚 tal que 1 = 2𝑚 es decir, 1 es
par, lo cual, es una contradicción, por tanto, 𝑛 es impar ∎

Demostración por contrarrecíproca


La demostración de un teorema 𝑝 → 𝑞 se dice que es por la contrarecíproca cuando
suponiendo que la conclusión, 𝑞, es falsa y utilizando la hipótesis 𝑝 y otros teoremas y
equivalencias lógicas establecidas previamente, se concluye que 𝑝 es falso.

Demostración por contraejemplos


Este tipo de demostración, está íntimamente relacionada con el cuantificador universal,
aparece cuando se quiere probar que una proposición del tipo ∀𝑥, 𝑝(𝑥) es falsa. Se dice que
se refuta la proposición ∀𝑥, 𝑝(𝑥).

Proposición 2.9
La suma de dos enteros positivos que sean cuadrados perfectos es también un cuadrado
perfecto.

Demostración
Evidentemente, 25 es un entero cuadrado perfecto y 36 es un entero cuadrado perfecto, pero
25+36=61 no es cuadrado perfecto, luego, la proposición que se pide demostrar no es cierta.

En ocasiones es necesario usar varios métodos para realizar una demostración.

24
Teorema 2.9 (Teorema fundamental de la aritmética)
Todo número natural compuesto 𝑛 > 1 se puede factorizar de manera única como 𝑛 =
𝛽 𝛽 𝛽
𝑝1 1 𝑝2 2 … . . 𝑝𝑘 𝑘 . Donde,
𝑝1 , 𝑝2 , … . , 𝑝𝑘 son primos distintos y 𝛽1 , 𝛽2 , … . , 𝛽𝑘 son enteros positivos.

Demostración
El resultado es cierto para n = 2.
Supongamos ahora que el resultado es cierto para
𝑛 = 3, 4, . . . , 𝑘. Hay que probar que es cierto para 𝑘 + 1.
Si 𝑘 + 1 es primo, listo.
Si 𝑘 + 1 es compuesto, entonces existen
𝑎, 𝑏 ∈ ℤ ; 1 < 𝑎 ≤ 𝑏 < 𝑘 + 1, tal que 𝑘 + 1 = 𝑎𝑏.
Pero, por hipótesis de inducción 𝑎 y 𝑏 se factorizan como producto de primos, así que 𝑘 + 1
también factoriza como producto de primos, a saber, los factores de 𝑎 y 𝑏. Supongamos, ahora,
que
𝑛 = 𝑟1 𝑟2 𝑠𝑟𝑢 = 𝑞1 𝑞2 𝑠𝑞𝑣
donde todos los 𝑟𝑖 𝑠 y los 𝑞𝑗 𝑠 son primos,
𝑟1 ≤ 𝑟2 ≤ ⋯ . ≤ 𝑟𝑢
𝑦
𝑞1 ≤ 𝑞2 ≤ ⋯ . ≤ 𝑞𝑣
Si cancelamos los primos iguales que hay en ambos lados nos queda
𝑟𝑖1 𝑟𝑖2 𝑠𝑟𝑖𝑢 = 𝑞𝑗1 𝑞𝑗2 𝑠𝑞𝑗𝑚 (todos distintos), entonces,
𝑟𝑖1 (𝑟𝑖2 𝑠𝑟𝑖𝑢 ) = 𝑞𝑗1 𝑞𝑗2 ∗ 𝑠𝑞𝑗𝑚
es decir, 𝑟𝑖1 divide a 𝑞𝑗1 𝑞𝑗2 𝑠𝑞𝑗𝑚 , entonces por el lema de Euclides, 𝑟𝑖1 divide a algún 𝑞𝑗 𝑡 ,
por tanto 𝑟𝑖1 = 𝑞𝑗 𝑡 , lo cual es una Contradicción ∎

Ejercicios 2.2
1) Suponga que hay 6 estudiantes en el club de matemáticas. Demuestre que 3 de ellos se
conocen entre sí o bien 3 de ellos no se conocen entre sí.
2) Demostrar que √𝑛 es irracional siempre que 𝑛 no sea cuadrado perfecto.
3) Demostrar que Si 𝑛 ∈ ℕ y 𝑛 ≠ 0 entonces 𝑛 ≥ 1.
4) Demostrar que para todo 𝑛 ∈ ℕ, 𝑛 < 𝑛+ .
5) Si 𝑚, 𝑛 ∈ ℕ con 𝑚 < 𝑛 y 𝑛 < 𝑟 entonces 𝑚 < 𝑟.
6) Si 𝑛 2 𝑁 con m < n entonces para todo p 2 N, m + p < n + p.
7) Si n ∈ ℕ con 𝑚 < 𝑛 entonces para todo 𝑝 ≠ 0, 𝑚𝑝 < 𝑛𝑝.
8) Si 𝑚, 𝑛 ∈ ℕ son tales que 𝑚 < 𝑛 entonces 𝑚+ ≤ 𝑛.
9) Si 𝑚, 𝑛, 𝑘 ∈ ℕ son tales que 𝑚 < 𝑛2 entonces 𝑚 ≤ 𝑛.
10) Si 𝑚, 𝑛, 𝑘 ∈ ℕ son tales que 𝑚𝑘 = 𝑛𝑘 y 𝑘 ≠ 0 entonces 𝑚 = 𝑛.

25
Unidad III: Esquemas Conceptuales.

Mapas conceptuales y aplicaciones.

Un mapa mental o mapa conceptual es un diagrama que representa ideas, palabras, y conceptos a
partir de una idea central. La principal función de los mapas mentales es poner en orden y potenciar
el flujo de información entre nuestro cerebro y el exterior, conectando de manera semántica y
jerárquica todo el conjunto de ideas.

El sistema de representación jerárquico fue inventado por el estagirita Aristóteles; sin embargo,
fue hasta 1971 que el psicólogo inglés Tony Buzan, basado en ideas de Leonardo Da Vinci,
formalizó el concepto de mapa mental. En sus propias palabras: "El Mapa Mental es la expresión
del pensamiento irradiante y, por tanto, una función natural de la mente. Es una técnica gráfica que
nos ofrece una llave maestra para acceder al potencial de nuestro cerebro. Se puede aplicar a todos
los aspectos de la vida, de modo que una mejoría en el aprendizaje y una mayor claridad de
pensamiento pueden reforzar el trabajo del hombre".

Si quiere saber más lea el libro Cómo crear mapas mentales de Tony Buzan. "Dedicado al cerebro
humano y a sus increíbles poderes de imaginación y asociación, ambos desarrollados y potenciados
mediante la magia de los Mapas Mentales".

A continuación, enumeramos algunas aplicaciones o programas para hacer mapas mentales o


mapas conceptuales.

1. Bubble.us. Es un sistema online gratuito donde facilitan la creación de diagramas o mapas


del tipo brainstorm (lluvia de ideas). Su servicio posibilita el trabajo colaborativo, la opción
de embeber o incrustar los diagramas en cualquier web, y la exportación de mapas mentales
en formato de imagen.

2. MindMeister. Una de las más completas herramientas online construida bajo el concepto
web 2.0. Posee diversas funcionalidades (trabajo colaborativo en tiempo real, adición de
notas, valoración de mapas mentales, historial de versiones, clasificación mediante
etiquetas, incrustación de iconos, etc). Además de la versión básica gratuita también cuenta
con una versión Premium pagada.

3. Mindomo. Otra aplicación en línea para la creación de cartografías mentales. Dispone de


una versión gratuita y otra comercial. Igual que la mayoría permite crear, editar y compartir
nuestros mapas mentales con colegas y amigos.

4. Mind42. Sin duda este servicio web tiene una de las interfaces más depuradas. Entre las
funciones que lo distinguen de sus antecesores están la incrustación de imágenes mediante
un buscador local y la vista previa de enlaces web adheridos en el texto.

26
5. Wisemapping. Un sitio web gratuito que combina la potencialidad de tecnología vectorial
con el dinamismo colaborativo de la web 2.0. Básicamente consta con todas las utilidades
mencionas en servicios anteriores.

6. Text 2 Mind Map. Aplicación online gratuita para crear mapas conceptuales. Cuenta con
funcionalidades de pago para administrar y exportar los mapas sin publicidad.

7. Litpen es una aplicación web para crear mapas conceptuales con forma de exágonos. Fue
lanzada en 2013 por Jesús Noland, Michelle Noland y Rachel Simanjuntak.

8. Freemind. es la primera aplicación gratuita escrita en Java para la creación de mapas


mentales. Permite generar mapas mentales en formato HTML y posteriormente publicarlos
en un sitio web o wiki. Se instala en cualquier sistema operativo y los requerimientos
técnicos mínimos de funcionamiento son Java Runtime Environment 1.4+, 13 MB de
espacio libre en disco duro y 512 MB de memoria RAM.

9. Semantik. Aplicación local exclusiva para el entorno de escritorio KDE (Linux). Esta
herramienta, orientada principalmente a estudiantes, bien podría auxiliar a cualquier
público que necesite crear documentos de forma rápida y sencilla: presentaciones, minutas,
disertaciones, tesis, informes.

10. RecallPlus. Software dirigido (sólo Windows) a estudiantes que exigen aprender más
rápido y mejorar su desempeño escolar. Fue escrito en 2001 con el objetivo de asistir a un
médico en su examen de medicina. Este software de estudio se divide en dos categorías:

1) Visual organizer/concept mapping/presentation software, es decir la organización y


presentación gráfica de ideas.

2) Revision/Flashcard software, una revisión a través de preguntas y respuestas estructuradas.

11. Labyrinth. Sencilla y ligera herramienta para el entorno de escritorio GNOME (Linux) con
las funciones esenciales de edición y gestión de mapas mentales: atributos de texto (negrita,
itálica, subrayado), exportación a formatos de imagen SVG, PNG y JPEG. Soporta
múltiples idiomas.

12. Vym (View Your Mind). Aplicación que genera y manipula sencillos mapas conceptuales
ideales para la gestión de tiempo y organización de tareas. Útil si se requiere obtener un
panorama más complejo y detallado de distintas ideas que ronden nuestra cabeza.
Únicamente se instala en plataformas Mac Os X y Linux.

13. PersonalBrain. Es una aplicación multiplataforma con una versión gratuita y otra de pago.
El valor añadido de este software radica en la inclusión de un calendario con el cual
podemos planificar cronológicamente nuestros mapas mentales.

27
14. Coogle. Herramienta para hacer mapas conceptuales o mentales de manera individual o
colaborativa. Cuenta con una versión gratuita y de pago. Además de estar disponible desde
la web, también hay versiones para iOS y Android.

15. GoConqr_examtimeGoconqr. Una de las herramientas más populares para crear mapas
mentales online. Cuenta con una aplicación móvil para visualizar todos los recursos que
proporciona Goconqr (mapas mentales, fichas, tests, diapositivas y apuntes). Disponible
para iOS y Android.

16. MAPMYself. Una sencilla opción para crear mapas mentales online. Cuenta con una
versión gratuita y otra de pago. Su utilización requiere tener instalado en el navegador
Silverlight de Microsoft.
17. XMind. Plataforma multilenguaje para crear mapas mentales desde el ordenador. Cuenta
con una versión gratuita para uso personal y otra versión profesional para empresas.
Disponible en Windows, Mac Os y Linux. (vía Xavi Aznar)

18. MindNode. Una de las mejores opciones para los usuarios de Mac Os (de pago). Cuenta
con una versión para escritorio y otra para dispositivos touch (IPod y Ipad).

19. Mindmanager (Windows/Mac/iOS) es algo más que un software de mapas mentales. Es


una suite completa de aplicaciones pensada para realizar brainstormings en grupo o
colaborar en proyectos. En realidad, es más un gestor de proyectos y una aplicación de
colaboración, pero tiene herramientas de mapas mentales muy potentes. Permite organizar
proyectos, asignar tareas, o determinar funciones muy específicas, y es igual de efectivo
para proyectos de cien personas, que para una docena o simplemente nuestras ideas
personales. Además, se integra con los servicios web y las herramientas que ya utiliza,
como Microsoft Office, Box, net y más. Cualquier persona familiarizada con la vieja
herramienta de MindManager estará encantada de saber que ha sido integrada como
aplicación individual de Mindjet. La mayoría de los usuarios de Mindjet son compañías
que están dispuestas a pagar por ello.

20. Cmaptools. Este software gratuito (versión off-line, on-line y en App) fue desarrollado en
el Institute for Human and Machine Cognition (IHMC) de la Universidad de West Florida.
Si bien la aplicación se hizo originalmente para mapas conceptuales, también pueden
realizarse con él los tradicionales mind maps. Es bastante intuitivo, amigable y fácil de
utilizar.

21. Imindmap. Es un software de pago, aunque dispone de una App gratuita con funciones
limitadas. Se fundamenta en la teoría de pensamiento irradiante de Tony Buzan. La
aplicación ya va por la versión 9, destacando por su versatilidad y atractivo diseño.

22. Visual Understanding Environment (VUE) (Open source, Mac, Windows, Linux). Es un
proyecto de la Tufts University que se focaliza en crear herramientas flexibles para generar
e integrar recursos digitales de apoyo a la enseñanza, el aprendizaje y la investigación.

23. Framindmap. Aplicación en francés similar a wisemapping.

28
24. Freeplane. Es una extensión basada en FreeMind que incorpora un mayor número de
funcionalidades para gestión del conocimiento e incluso para hacer brainstorming. Su
interfaz es más dinámica, y se puede tomar notas que se van agregando a los nodos, a la
vez que permite organizar ideas en diferentes jerarquías, clasificar nodos a través de
atributos en forma de metadatos y aplicar diferentes estilos para que el mapa tenga un
aspecto más atractivo.

25. Calligra Braindump (Open source, Mac, Windows, Linux). Las mismas funcionalidades
básicas de cajas/formas para conceptos, líneas de relaciones (con trazos que usa
herramientas de dibujo), no tan intuitiva y con algunos problemas a juzgar por los foros o
detección de errores en el funcionamiento.

26. Mind Mup Aplicación on-line para trabajar colaborativamente, pantalla fácil de usar con
funciones comunes (modificación de grosor de trazo, tipo de fuente, color, enlaces),
permite importar y exportar mapas en diferentes formatos. La versión gold permite alojar
mapas en una biblioteca.

27. Mapul o Map Myself (on-line). Cuenta con una de las interfaces más atractivas, en mi
opinión de las mejores. La versión gratuita es suficiente para usos educativos y hasta
profesionales. Posee un diseño orgánico donde las formas de las ramas, color, grosor de
líneas, etc. se diferencian de la mayoría de aplicaciones disponibles. Está en cinco idiomas
(inglés, español, japonés, checo y ruso).

28. Blumind (Windows). Software para un diseño básico de mapas mentales, con una interfaz
sencilla y fácil, edición de documentos múltiples, soporte para inglés y chino, también
compatible para diseñar organigramas, diagramas de árbol, diagramas lógicos, etc. Soporta
múltiples formatos de exportación, incluyendo todos los formatos comunes de imagen y
PDF, SVG, texto, etc.

29. Creately. Herramienta que incluye también la posibilidad de hacer diagramas de flujo y
otro tipo de organigramas, facilita la participación entre los usuarios, tanto en educación
como en empresas. Versión off-line y en la nube, dispone de tutoriales y blog y mies de
plantillas de ejemplos.

30. Personal brain (todos los formatos). Aplicación sencilla que admite diferentes
funcionalidades que van desde hacer diagramas y mapas pero que se ofrece como un
servicio de almacenamiento y gestión de la información y el conocimiento, ya sea
individual o en organizaciones. Solicita registro y tiene dos precios para usuario individual
dependiendo de la versión.

29
Ejercicios 3.1
o En grupos de 2 o 3 personas discutir las características del siguiente mapa mental.

o En grupos de dos o tres personas escoger una de las aplicaciones para crear mapas mentales
y diseñar un mapa mental en el cual el concepto central es “Lógica” y presenta todo lo visto
hasta este momento.

o Presente su mapa mental impreso y en digital en una puesta en común.

30
Diagramas de flujos.

Los diagramas de flujo usan formas conectadas por líneas para representar los diferentes pasos en
un proceso. Visualizando el proceso se pueden identificar errores y optimizarlo.

Ejemplos

31
Ejercicios 3.2
Para cada problema siguiente, use SmartDraw para diseñar un diagrama de flujos para un programa
que lo resuelva
1. Crear una tabla que convierta pulgadas a pies.
2. Determinar la hipotenusa de un triángulo rectángulo
3. Determinar el número de formaciones, el usuario introducirá la cantidad de elementos que
dispone, la cantidad de elementos que entra en cada formación y la incidencia del orden de los
elementos en dichas formaciones.

32
Unidad IV: Resolución De Problemas.

En esta unidad haremos uso de las notas preparadas y presentadas por el Dr. José Nieto Said en el
marco de las Olimpiadas Centroamericanas y del Caribe de 2010. Las resumimos a continuación
para fines didácticos.

La resolución de problemas es considerada la más compleja de las funciones intelectuales y ha


sido definida como un proceso que requiere el control y manejo de otras habilidades fundamentales
como el análisis, la deducción, la creatividad y la memoria. Este proceso inicia cuando una persona
desea llegar a un objetivo, pero no sabe exactamente cómo proceder. Se llama solución de un
problema, a la forma de llegar al objetivo deseado. Cuando la persona sabe cómo proceder para
llegar al objetivo, la situación no es considerada un problema, sino un ejercicio y lo que procede
es “simplemente” seguir los pasos para llegar a este objetivo.

Aunque los ejercicios forman parte fundamental en el proceso de aprendizaje de la matemática, no


será este el énfasis aquí. Por ejemplo, se quiere encontrar la suma de todos los números impares
menores que 2, 000,000. Obtener esa suma usando lápiz y papel (sin usar calculadora, hoja de
cálculo u otro instrumento), es para muchas personas un problema.

De esta forma, una situación puede representar un problema para algunos y no serlo para otros,
dependiendo de la experiencia de cada uno. Podemos decir entonces que un problema se resuelve
una sola vez, después se convierte en ejercicio, así como cualquier otra situación parecida. Una
consecuencia importante de esto es que cada problema debe intentarse durante un tiempo suficiente
antes de revisar la solución.

La resolución de problemas está estrechamente relacionada con la creatividad, que algunos definen
precisamente como la habilidad para generar nuevas ideas y solucionar todo tipo de problemas y
desafíos.

Resolver un problema es hacer un descubrimiento. Un gran problema significa un gran


descubrimiento, pero hay una partícula de descubrimiento en la solución de cualquier problema.
El suyo puede ser modesto, pero si pone a prueba la curiosidad que induce a poner en juego las
facultades inventivas, y si lo resuelve por medios propios, puede experimentar la tensión y el
encanto del descubrimiento y el goce del triunfo.

Todo ser humano nace con un gran potencial para la creación, pero mientras algunos lo aprovechan
al máximo, otros casi no lo utilizan. La creatividad, al igual que cualquier otra habilidad humana,
puede desarrollarse a través de la práctica y el entrenamiento adecuado. Lamentablemente,
también puede atrofiarse si no se ejercita de forma adecuada.

El pensamiento se ha dividido en divergente y convergente. El primero consiste en la habilidad


para pensar de manera original y elaborar nuevas ideas, mientras que el segundo se relaciona con
la capacidad crítica y lógica para evaluar alternativas y seleccionar la más apropiada. Estos dos
tipos de pensamiento se asocian a los hemisferios cerebrales derecho e izquierdo, respectivamente.

33
En efecto, la neurociencia ha establecido la especialización de los hemisferios cerebrales: la
capacidad de hablar, escribir, leer y razonar con números, es fundamentalmente una
responsabilidad del hemisferio izquierdo; mientras que la habilidad para percibir y orientarse en
el espacio, trabajar con figuras geométricas y rotarlas mentalmente, son ejecutadas
predominantemente por el hemisferio derecho. El hemisferio derecho sería también el asiento de
las emociones y de la creatividad artística, mientras que al izquierdo le corresponden
los procesos cognitivos en los cuales el lenguaje y la lógica juegan un rol central. Ambos tipos de
pensamiento juegan un rol fundamental en la resolución de problemas, que por consiguiente debe
ser abordada, por así decirlo, con el cerebro completo.

Técnicas generales

Invertir el problema
Cada concepto tiene uno contrario y la oposición entre ellos genera una tensión favorable al hecho
creativo. Esta idea, que tiene profundas raíces tanto en la filosofía oriental como en la occidental,
se refleja en la sabiduría popular en aforismos tales como: “Para saber mandar hay que aprender a
obedecer” o “Para ser un buen orador hay que saber escuchar”.

Como ejemplo de esta técnica supongamos que se desea diseñar un zapato que sea muy cómodo.
El problema inverso sería diseñar un zapato incómodo. Esto puede parecer absurdo, pero el análisis
de este problema nos permitirá posiblemente descubrir los factores que causan incomodidad en un
zapato, y al evitarlos se habrá dado un gran paso hacia la solución del problema original.

En matemática el pensamiento inverso interviene en las demostraciones “por reducción al


absurdo”, en las cuales se toma como punto de partida precisamente lo contrario a lo que se quiere
probar, para tratar de llegar a una contradicción.

Pensamiento lateral
Consiste en explorar alternativas inusuales o incluso aparentemente absurdas para
resolver un problema. En otras palabras: evitar los caminos trillados, intentar lo que
nadie ha intentado, ensayar percepciones y puntos de vista diferentes

Los problemas realmente difíciles, aquellos que resisten los esfuerzos de los mejores
solucionadores, requieren frecuentemente un enfoque de este tipo para ser resueltos. Muchos
problemas matemáticos se resuelven de manera sorprendente estableciendo conexiones
inesperadas con ramas de la matemática que aparentemente no guardan relación alguna con el
problema original.

Principio de discontinuidad
La rutina suprime los estímulos necesarios para el acto creativo, por lo tanto, si experimenta un
bloqueo temporal de su capacidad creadora, interrumpa su programa cotidiano de actividades y
haga algo diferente a lo acostumbrado. Vaya a dar un paseo por sitios que no conoce, ensaye una
nueva receta de cocina, escuche música diferente a la que escucha habitualmente, lea un libro que
no tenía pensado leer, asista a algún tipo de espectáculo diferente a sus favoritos. Romper la rutina
suele provocar una liberación de energía creadora suficiente para resolver problemas que se nos
han resistido durante mucho tiempo.

34
Imitación
La mayor parte de los grandes artistas comienzan imitando a sus maestros. Más aún se ha llegado
a afirmar, en parte en broma y en parte en serio, que “la originalidad no es otra cosa que un plagio
no detectado”. En cualquier caso, es claro que la imitación puede ser un primer paso válido hacia
la originalidad, ya que permite consolidar una técnica sobre la cual puede luego edificarse algo
novedoso. Si usted desea convertirse en un solucionador experto, no vacile en observar e imitar
las técnicas de resolución de problemas empleadas con éxito por sus compañeros, maestros o
colegas.

Factores emocionales
La resolución de problemas no es un asunto puramente intelectual. Las emociones, y en particular
el deseo de resolver un problema, tienen también una gran importancia. La incapacidad que
manifiestan algunos alumnos para resolver incluso el ejercicio más sencillo no es producto por lo
general de una deficiencia intelectual, sino de una absoluta falta de interés y motivación. A veces
no existe ni siquiera el deseo de comprender el problema, y por lo tanto el mismo no es
comprendido.

El profesor que desee ayudar realmente a un alumno con estas características deberá ante todo
despertar su curiosidad dormida, motivarlo y transmitirle deseos de logro y superación. Algunas
creencias negativas para el proceso creativo están asociadas a una baja autoestima y pueden tener
raíces emocionales profundas. Por ejemplo, hay quienes enfrentados a un problema creen a priori
que no podrán resolverlo, y que si lo intentan sólo conseguirán terminar con un dolor de cabeza.
El maestro o profesor debe en estos casos apelar a todas sus dotes y conocimientos como educador,
aunque en casos extremos será necesaria también la ayuda de un orientador o la de un psicólogo.

En el polo opuesto, alguien que tenga confianza en su propia capacidad, y crea que un problema
es un desafío que vale la pena enfrentar y que resolverlo le proporcionará una satisfacción
intelectual al mismo tiempo que será una experiencia valiosa para su formación, estará en
excelentes condiciones psicológicas para abordar el proceso de resolución del problema.

Bloqueos mentales
Los bloqueos mentales son barreras que nos impiden percibir un problema en la forma correcta y
encontrarle solución. Se pueden clasificar como:

Bloqueos perceptivos: estereotipos, dificultad para aislar el problema, delimitar demasiado el


espacio de soluciones, imposibilidad de ver el problema desde varios puntos de vista, saturación,
no poder utilizar toda la información sensorial.

Bloqueos emocionales: miedo a cometer errores, a arriesgar, a fracasar; deseo de seguridad y


orden; preferir juzgar ideas a concebirlas; inhabilidad para relajarse; falta de estímulo; entusiasmo
excesivo; falta de control imaginativo. Bloqueos culturales: tabúes; el peso de la tradición; roles
predeterminados asignados a la mujer y al hombre.

Bloqueos ambientales: distracciones; falta de apoyo para llevar adelante una idea; falta de
cooperación entre colegas.

35
Bloqueos intelectuales: inhabilidad para seleccionar un lenguaje apropiado para el problema
(verbal, matemático, visual); uso inadecuado de las estrategias; falta de información o información
incorrecta.

Bloqueos expresivos: técnicas inadecuadas para registrar y expresar ideas (a los demás y a uno
mismo).

Si un problema se resiste a nuestros mejores esfuerzos, nos queda todavía la posibilidad de dejarlo
durante un tiempo, descansar, dar un paseo, y volver a él más tarde. Sin embargo, solamente
aquellos problemas que nos han apasionado, manteniéndonos en una considerable tensión mental,
son los que vuelven más tarde, transformados, a la mente consciente. La inspiración o iluminación
súbita, que los antiguos consideraban un don divino, hay que merecerla.

A continuación, se presentan algunas pautas para abordar un problema en busca de resolverlo.

i. La metodología de Pólya
En 1945, el insigne matemático y educador George Pólya (1887–1985) publicó un libro que
rápidamente se convertiría en un clásico: How to solve it (Cómo resolverlo). En el mismo propone
una metodología en cuatro etapas para resolver problemas. A cada etapa le asocia una serie de
preguntas y sugerencias que, aplicadas adecuadamente, ayudarán a resolver el problema. Las
cuatro etapas y las preguntas a ellas asociadas se detallan a continuación:

Etapa I: Comprensión del problema.


¿Cuál es la incógnita?
¿Cuáles son los datos?
¿Cuál es la condición?
¿Es la condición suficiente para determinar la incógnita?
¿Es insuficiente?
¿Redundante?
¿Contradictoria?

Etapa II: Concepción de un plan.


¿Se ha encontrado con un problema semejante?
¿Ha visto el mismo problema
planteado en forma ligeramente diferente?
¿Conoce un problema relacionado con éste?
¿Conoce algún teorema que le pueda ser útil?
“Mire atentamente la incógnita y trate de recordar un problema que le sea familiar y que tenga la
misma incógnita o una incógnita similar. He aquí un problema relacionado con el suyo y que se ha
resuelto ya”.
¿Podría utilizarlo?
¿Podría emplear su resultado?
¿Podría utilizar su método?
¿Podría utilizarlo introduciendo algún elemento auxiliar?
¿Podría enunciar el problema en otra forma?

36
¿Podría plantearlo en forma diferente nuevamente?
“Refiérase a las definiciones. Si no puede resolver el problema propuesto, trate de resolver primero
algún problema similar”.
¿Podría imaginarse un problema análogo un tanto más accesible?
¿Un problema más general?
¿Un problema más particular?
¿Un problema análogo?
¿Puede resolver una parte del problema? “Considere sólo una parte de la condición”.
¿En qué medida la incógnita queda ahora determinada?,
¿En qué forma puede variar?
¿Puede usted deducir algún elemento útil de los datos?
¿Puede pensar en algunos otros datos apropiados para determinar la incógnita?
¿Puede cambiar la incógnita?
¿Puede cambiar la incógnita o los datos, o ambos si es necesario, de tal forma que la nueva
incógnita y los nuevos datos estén más cercanos entre sí?
¿Ha empleado todos los datos?
¿Ha empleado toda la condición?
¿Ha considerado usted todas las nociones esenciales concernientes al problema?

Etapa III: Ejecución del plan.


Al ejecutar el plan, compruebe cada uno de los pasos.
¿Puede ver claramente que el paso es correcto?
¿Puede demostrarlo?

Etapa IV. Visión retrospectiva.


¿Puede usted verificar el resultado?
¿Puede verificar el razonamiento?
¿Puede obtener el resultado en forma diferente?
¿Puede verlo de golpe?
¿Puede emplear el resultado o el método en algún otro problema?

La primera etapa es insoslayable: es imposible resolver un problema del cual no se comprende el


enunciado. Sin embargo, hemos visto a muchos estudiantes lanzarse a efectuar operaciones y
aplicar fórmulas sin reflexionar siquiera un instante sobre lo que se les pide. Por ejemplo, si en el
problema aparece una función, comienzan de inmediato a calcularle la derivada,
independientemente de lo que diga el enunciado. Si el problema se plantea en un examen y luego,
comentando los resultados, el profesor dice que el cálculo de la derivada no se pedía y más aún
que el mismo era irrelevante para la solución del problema, algunos le responderán: ¿o sea que no
nos va a dar ningún punto por haber calculado la derivada? Este tipo de respuesta revela una
incomprensión absoluta de lo que es un problema y plantea una situación muy difícil al profesor,
quien tendrá que luchar contra vicios de pensamiento arraigados, adquiridos tal vez a lo largo de
muchos años.

La segunda etapa corresponde a la estrategia, es decir a la formulación de un plan general para


atacar y resolver el problema, dejando los detalles técnicos de su ejecución para un momento
posterior.

37
Las preguntas que Pólya asocia a esta etapa están dirigidas a posibilitar la elaboración de un plan
factible, por ejemplo, llevando el problema hacia un terreno conocido. Esta etapa es la más sutil y
delicada, ya que no solamente está relacionada con los conocimientos y la esfera de lo racional,
sino también con la imaginación y la creatividad. Las recomendaciones de Pólya son muy útiles
para el tipo de problemas que suele plantearse en los cursos ordinarios de matemática, sin embargo,
se quedan un poco cortas para problemas olímpicos verdaderamente originales.

La tercera etapa corresponde a la táctica, es decir a los recursos técnicos necesarios para ejecutar
con éxito el plan estratégico. Si el plan está bien concebido, es factible y se poseen los
conocimientos y el entrenamiento necesarios, debería ser posible ejecutarlo sin contratiempos. Sin
embargo, por lo general en esta etapa se encontrarán dificultades que nos obligarán a regresar a la
etapa anterior para realizar ajustes al plan o incluso para modificarlo por completo, una o más
veces.

La cuarta etapa es muchas veces omitida, incluso por solucionadores expertos. Pero Pólya insiste
mucho en su importancia, en primer lugar, porque comprobar los pasos realizados y verificar su
corrección nos puede ahorrar muchas sorpresas desagradables. Desconfíe de una solución hallada
fácilmente y sin esfuerzo alguno.

Pero la razón más importante para practicar la visión retrospectiva es que puede conducir a nuevos
resultados que generalicen, amplíen o fortalezcan el que se acaba de hallar. Henri Poincaré decía
que “no hay problemas resueltos, sólo hay problemas más o menos resueltos”. En particular, la
visión retrospectiva puede permitirle generar nuevos problemas a partir del que acaba de resolver.

ii. El aporte de Alan Schoenfeld


Si bien la mayoría de los matemáticos reconocen en las estrategias heurísticas de Pólya los métodos
que ellos mismos utilizan habitualmente, no es tan fácil para el que no tiene experiencia aplicarlas
exitosamente. En otras palabras, dichas estrategias son más descriptivas que prescriptivas. Alan
Schoenfeld es uno de los que más han estudiado esta problemática. En su análisis identifica los
siguientes cuatro factores relevantes para la resolución de problemas:

Recursos cognitivos.
Son nuestros conocimientos matemáticos generales, tanto de conceptos y resultados como de
procedimientos (algoritmos).

Heurística.
Es el conjunto de estrategias y técnicas para resolver problemas que conocemos y estamos en
capacidad de aplicar.

Control o metacognición. Es la capacidad de utilizar lo que sabemos para lograr un objetivo.

Creencias.
Se refiere a aquellas creencias y opiniones relacionadas con la resolución de problemas y que
pueden afectarla favorable o desfavorablemente.

38
La importancia del primer factor es obvia. Sin embargo, no es suficiente poseer un amplio bagaje
de conocimientos matemáticos para ser un solucionador experto. También es necesario dominar
algunas técnicas y estrategias que nos ayuden a atacar el problema. En dominios restringidos y
bien delimitados, en los cuales los problemas a resolver son más o menos rutinarios, se han
desarrollado estrategias que pueden ser aplicadas con éxito incluso por un computador, con
resultados tan buenos o mejores que los obtenidos por los expertos humanos (estos son los famosos
sistemas expertos, producto de las investigaciones en inteligencia artificial y ciencia cognitiva).
No obstante, para resolver problemas no rutinarios en dominios ricos en contenido, como la
matemática, se requiere algo más que conocimientos y estrategias. Ese factor adicional es lo que
llamamos control; actúa como una voz interior que nos dice qué ideas y estrategias (entre muchas
alternativas posibles) nos conviene aplicar para el problema que tenemos entre manos, o bien si
debemos abandonar un camino que no parece arrojar resultados o por el contrario redoblar
esfuerzos y perseverar en él. Los solucionadores inexpertos tienen evidentes deficiencias en este
aspecto:
se apresuran a transitar el primer camino que se les ocurre y luego se mueven en círculos, cayendo
una y otra vez en el mismo error.
El último factor puede influir también de manera importante en el proceso de resolución de
problemas. Algunas creencias comunes, sobre todo entre estudiantes de enseñanza media, son las
siguientes: “todo problema se resuelve mediante alguna fórmula”, “lo importante es el resultado y
no el procedimiento”, “la respuesta del libro no puede estar equivocada”. Este tipo de creencias es
un obstáculo para el desempeño de cualquier persona como solucionador.

Schoenfeld elaboró también una lista de las estrategias más utilizadas:


1. Análisis.
• Dibuje un diagrama siempre que sea posible.
• Examine casos especiales.
• Seleccione algunos valores especiales para ejemplificar el problema e irse familiarizando
con él.
• Examine casos límite para explorar el rango de posibilidades.
• Si hay un parámetro entero positivo, dele sucesivamente los valores 1, 2, . . . , 𝑚 y vea si
emerge algún patrón inductivo.
• Trate de simplificar el problema, explotando la existencia de simetría, usando argumentos
del tipo “sin pérdida de generalidad”.

2. Exploración.
• Considere problemas esencialmente equivalentes.
• Reemplazando condiciones por otras equivalentes.
• Recombinando los elementos del problema de maneras diferentes.
• Introduciendo elementos auxiliares.
• Reformulando el problema:
o Mediante un cambio de perspectiva o notación.
o Mediante argumentos por contradicción o contraposición.
o Asumiendo que tenemos una solución y determinando sus propiedades.
• Considere un problema ligeramente modificado.
• Escoja submetas (tratando de satisfacer parcialmente las condiciones).
• Relaje una condición y luego trate de reimponerla.
39
• Descomponga el dominio del problema y trabaje caso por caso.
• Considere problemas sustancialmente modificados.
• Construya un problema análogo con menos variables.
• Deje todas las variables fijas excepto una, para determinar su impacto.
• Trate de aprovechar cualquier problema relacionado que tenga forma, datos o conclusiones
similares.

3. Verificación de la solución.
• ¿Pasa su solución estas pruebas específicas?
o ¿Usa todos los datos pertinentes?
o ¿Está de acuerdo con estimaciones o predicciones razonables?
o ¿Soporta pruebas de simetría, análisis dimensional y escala?
• ¿Pasa estas pruebas generales?
o ¿Puede ser obtenida de manera diferente?
o ¿Puede ser sustanciada por casos especiales?
o ¿Puede ser reducida a resultados conocidos?
o ¿Puede utilizarse para generar algún resultado conocido?

iii. MODELO DE MASOM-BURTON-STACEY


En este modelo sólo se consideran tres fases
• Abordaje (son 1ra y la 2da de Polya).
• Ataque (llevar a cabo el plan.
• Revisión (analizar si la solución obtenida es correcta y si se puede aplicar a otro problema).

iv. MODELO DE MIGUEL DE GUZMÁN


Este modelo consta de cuatro fases.
• Familiarización con el problema.
• Búsqueda de estrategia (tener un plan).
• Llevar adelante las ideas.
• Revisar el proceso y sacar consecuencias de él.

Ejemplo 4.1
Dividir cien panes entre cinco hombres, de modo que las porciones que reciban estén en
progresión aritmética y que la séptima parte de la suma de las tres mayores sea igual a la suma
de las dos porciones menores.

Solución.
Asegurémonos de comprender bien el problema.
1. ¿Qué se nos pide?
Dividir cien panes entre cinco hombres, de modo que se cumplan ciertas condiciones.
2. ¿Cuáles son los datos?
El número total de panes (100), la cantidad de porciones (5) y las condiciones que debe
cumplir el reparto.

40
3. ¿Cuáles son las incógnitas?
Obviamente, la cantidad de panes que le corresponderá a cada uno.
4. ¿Comprendemos la condición?
En primer lugar, las porciones deben estar en progresión aritmética; esto significa que, si
escribimos las porciones en orden creciente de magnitud, la diferencia de cada una de ellas
con la siguiente es constante. En otras palabras, si llamamos 𝑥 a la menor de las porciones y
𝑟 a la diferencia común (o razón) de la progresión, entonces las cinco porciones deberán ser
𝑥, 𝑥 + 𝑟, 𝑥 + 2𝑟, 𝑥 + 3𝑟 𝑦 𝑥 + 4𝑟. Utilizando esta notación podemos describir la última
condición del problema mediante una ecuación:
(𝑥 + 2𝑟) + (𝑥 + 3𝑟) + (𝑥 + 4𝑟)
= 𝑥 + (𝑥 + 𝑟). *
7
5. ¿Es la condición suficiente para determinar la incógnita?
6. ¿Es insuficiente? Estas preguntas vienen muy bien en este momento, ya que nos hacen
observar que tenemos dos incógnitas 𝑥 y 𝑟 pero una sola ecuación. En general (pero por
supuesto hay excepciones) esto significa que el problema es indeterminado, es decir que
en vez de una única solución admite varias, tal vez hasta un número infinito de ellas. Pero
otra posibilidad a tener en cuenta es que no tengamos suficientes ecuaciones sencillamente
por haber pasado por alto algún dato o condición del problema. Recordemos las preguntas
de Pólya:
7. ¿Ha empleado todos los datos?
8. ¿ha empleado toda la condición?
Bueno, leyendo una vez más el enunciado del problema vemos que no hemos utilizado el
hecho de que los panes a dividir son cien. Este dato nos permite escribir otra ecuación:
𝑥 + (𝑥 + 𝑟) + (𝑥 + 2𝑟) + (𝑥 + 3𝑟) + (𝑥 + 4𝑟) = 100. **
Bien, ya tenemos dos ecuaciones y dos incógnitas. El plan a seguir es simple: resolver el
sistema. Para ello simplificamos primero las ecuaciones * y ** hasta obtener el sistema:
11𝑥 − 2𝑟 = 0
{ }
𝑥 + 2𝑟 = 20
de donde resulta 𝑥 = 5/3 y 𝑟 = 55/6.
Las cinco porciones serán entonces:
5 2
=1
3 3
5 55 65 5
+ = = 10
3 6 6 6
65 55
+ = 20
6 6
1
20 + 55/6 = 175/6 = 29
6
y finalmente,
175 55 115 1
+ = = 38 .
6 6 3 3

41
Visión retrospectiva:
9. ¿Puede usted verificar el resultado?
Esto es fácil:
5/3 + 65/6 + 20 + 175/6 + 115/3 = 100
y
65/6 − 5/3 = 20 − 65/6 = 175/6 − 20 = 115/3 − 175/6 = 55/6.
10. ¿Puede obtener el resultado en forma diferente?
Bueno, si se tiene cierta experiencia resolviendo problemas con progresiones aritméticas se
observa que muchas veces resulta más cómodo representar la progresión de manera simétrica,
alrededor de un término central.
En nuestro caso, si llamamos 𝑧 al término central y 𝑟 a la diferencia común, los cinco términos
serán
➢ 𝑧 − 2𝑟
➢ 𝑧−𝑟
➢ 𝑧
➢ 𝑧+𝑟
➢ 𝑧 + 2𝑟
Ahora la condición de que las partes suman cien se escribe así:
(𝑧 − 2𝑟) + (𝑧 − 𝑟) + 𝑧 + (𝑧 + 𝑟) + (𝑧 + 2𝑟) = 100
que se reduce a
5𝑧 = 100
y por tanto
𝑧 = 20
(más en general, el término medio de una progresión aritmética con un número impar de
términos es igual al promedio de todos los términos). Si ahora llamamos 𝑆 a la suma de los
dos términos menores, la otra condición del problema nos dice que
𝑆 = (100 − 𝑆)/7.
De aquí se despeja
𝑆 = 25/2.
Como
𝑆 = 20 − 2𝑟 + 20 − 𝑟 = 40 − 3𝑟
se deduce
40 − 𝑆 55
𝑟 = = .
3 6
Se obtiene por supuesto la misma solución que antes, pero el procedimiento luce más
limpio y elegante: en lugar de resolver un sistema de dos ecuaciones con dos incógnitas se
resuelven ecuaciones de primer grado.
¿Cómo resolvían los egipcios este problema?
Bueno, eso lo dejaremos a la imaginación del lector, pero un indicio es que utilizaban el
método de la falsa posición, consistente en plantear una solución tentativa para luego

42
modificarla de modo que se cumplan las condiciones requeridas. Por ejemplo, para dividir
100 en dos partes tales que una sea un séptimo de la otra (cómo se hizo más arriba para hallar
la suma 𝑆 de los dos términos menores), se puede partir de 1 y 7.
Como 1 es un séptimo de 7, cumplen una de las condiciones, pero no la otra ya que suman 8.
Ahora bien, como multiplicando 8 por 25/2 se obtiene 100, multiplicando 1 y 7 por 25/2 se
obtienen 25/2 y 175/2, que suman 100 y están en la proporción de 1 a 7, ahora sólo queda
determinar la diferencia común 𝑟 para que los tres términos superiores
(20, 20 + 𝑟 y 20 + 2𝑟) sumen 175/2.
Para complicar aún más las cosas, tenga en cuenta que los egipcios no sólo no disponían del
álgebra simbólica, sino que tampoco manejaban las fracciones como nosotros estamos
acostumbrados a hacerlo. En efecto, sólo escribían fracciones con numerador 1, y cualquier
otra la escribían como suma de estas fracciones particulares. Por ejemplo 3/7 lo representaban
1 1 1
como + + .
3 7 231

Ejemplo 4.2
Determinar todos los números de tres cifras 𝑎𝑏𝑐 para los cuales 𝑎𝑏𝑐 = 𝑎! + 𝑏! + 𝑐!

Solución.
Este es un típico problema que se puede resolver mediante un examen de casos exhaustivo.
Estos problemas son más fáciles de resolver para una computadora que para un ser humano,
ya que la computadora puede proceder por fuerza bruta analizando todos los casos uno tras
otro rápidamente y sin errores. En cambio, un ser humano debe administrar mejor sus fuerzas,
delimitando cuidadosamente los casos que se van a estudiar. En este problema se puede
comenzar por observar que cada cifra debe ser menor o igual a 6, ya que si alguna fuese 7 o
más el miembro derecho superaría a 7! = 5040. Más aún ninguna puede ser 6, ya que de lo
contrario el miembro derecho superaría a 6! = 720 y a debería ser por lo menos 7, lo cual es
imposible.
En resumen, las posibles soluciones deben cumplir
0 ≤ 𝑎, 𝑏, 𝑐 ≤ 5
y
𝑎 ≠ 0.
Pero entonces
𝑎! + 𝑏! + 𝑐! ≤ 5! + 5! + 5! = 360
y vemos que
𝑎 ≤ 3.
Entonces
𝑎! + 𝑏! + 𝑐! ≤ 3! + 5! + 5! = 246
y vemos que
𝑎 ≤ 2.

43
Hay por lo tanto sólo dos posibilidades para 𝑎, que son 1 y 2. Analicemos cada uno de estos
casos por separado.
Caso 𝑎 = 1:
Debe cumplirse
100 + 10𝑏 + 𝑐 = 1 + 𝑏! + 𝑐!,
es decir
99 + 10𝑏 + 𝑐 = 𝑏! + 𝑐!
De aquí se deduce que 𝑏 o 𝑐 deben valer 5 (si no 𝑏! + 𝑐! no pasaría de 48).
Con 𝑏 = 5
se tiene
100 + 50 + 𝑐 = 1 + 120 + 𝑐!
o bien
29 + 𝑐 = 𝑐!
que claramente no se satisface para ningún 𝑐. Con
𝑐 = 5 𝑠𝑒 𝑡𝑖𝑒𝑛𝑒 100 + 10𝑏 + 5 = 1 + 𝑏! + 120
o bien
10𝑏 = 𝑏! + 16
que se satisface para 𝑏 = 4.
Así hemos hallado la solución 𝑎 = 1, 𝑏 = 4, 𝑐 = 5.

Caso 𝑎 = 2: debe cumplirse


200 + 10𝑏 + 𝑐 = 2 + 𝑏! + 𝑐!
es decir
198 + 10𝑏 + 𝑐 = 𝑏! + 𝑐!.
Para que esto se cumpla la única posibilidad es que sea
𝑏 = 𝑐 = 5
pues si uno de ellos fuese menor que 5 entonces
𝑏! + 𝑐! ≤ 144.
𝑎 = 2
Pero, { 𝑏 = 5 } no es solución, ya que
𝑐 = 5
2! + 5! + 5! = 242 6 = 255.
𝑎 = 1
En conclusión, hay una única solución: { 𝑏 = 4 }
𝑐 = 5

44
Ejemplo 4.3
¿En cuántas regiones queda dividido el plano por 6 rectas en posición genérica (es decir, tales
que no haya dos de ellas paralelas ni tres concurrentes en un punto)?

Solución.
Evidentemente una recta divide el plano en dos regiones, y dos rectas no paralelas lo dividen
en cuatro. Pero ya para tres rectas el problema comienza a complicarse. Si trazamos unos
cuantos diagramas veremos que la tercera recta atraviesa siempre a tres de las cuatro regiones
determinadas por las dos primeras, pero no a la cuarta, y por lo tanto la respuesta para tres
rectas parece ser siete. Pero,
1. ¿Podemos estar seguros de esto?
2. ¿Y qué pasará cuando tracemos la cuarta, la quinta y la sexta recta?
Lamentablemente, los dibujos se complican demasiado, algunas rectas se cortan fuera de la
hoja y no es fácil contar las regiones sin equivocarnos. Además, pareciera que la respuesta
depende de cómo dibujemos las rectas. Volvamos entonces al principio.
3. ¿Podría imaginarse un problema análogo un tanto más accesible?
Bueno, en vez de disminuir el número de rectas podemos disminuir la dimensión, es decir
considerar en cuántas regiones queda dividida una recta por cierto número de puntos. Este
problema sí es fácil, 𝑛 puntos dividen a la recta en 𝑛 + 1 regiones (a saber 𝑛 − 1 segmentos
y 2 semirrectas).
4. ¿Y no podemos aprovechar este resultado para el problema en el plano?
Veamos, si ya hemos trazado n − 1 rectas, entonces, al trazar la 𝑛 − é𝑠𝑖𝑚𝑎, ésta cortará a las
anteriores en 𝑛 − 1 puntos diferentes (por la hipótesis de genericidad).
Por tanto, la 𝑛 − é𝑠𝑖𝑚𝑎 recta quedará dividida en n partes por esos puntos de intersección.
Pero es claro que cada una de esas partes estará contenida por completo en una región de las
determinadas por las primeras n−1 rectas, región que quedará dividida en dos por la n-sima
recta. Por tanto, hemos descubierto que al trazar la n-sima recta el número de regiones aumenta
en n unidades. Apliquemos ahora este resultado desde el comienzo y de manera sucesiva.
Inicialmente hay una sola región: el plano.
Al trazar la primera recta el número de regiones aumenta en una unidad, y tendremos
1 + 1 = 2 𝑟𝑒𝑔𝑖𝑜𝑛𝑒𝑠.
Al trazar la segunda recta el número de regiones aumenta en dos unidades, y tendremos
2 + 2 = 4 𝑟𝑒𝑔𝑖𝑜𝑛𝑒𝑠.
Al trazar la tercera recta el número de regiones aumenta en tres unidades, y tendremos
4 + 3 = 7 𝑟𝑒𝑔𝑖𝑜𝑛𝑒𝑠.
Hasta aquí los resultados concuerdan con lo que ya sabíamos. Ahora resulta fácil continuar:
para cuatro rectas son
7 + 4 = 11 𝑟𝑒𝑔𝑖𝑜𝑛𝑒𝑠
para cinco rectas son
11 + 5 = 16 𝑟𝑒𝑔𝑖𝑜𝑛𝑒𝑠

45
para seis rectas son
16 + 6 = 22 𝑟𝑒𝑔𝑖𝑜𝑛𝑒𝑠.
Visión retrospectiva: Resulta natural preguntarse cuál será el número de regiones en que queda
dividido el plano por un número 𝑛 cualquiera de rectas en posición genérica. Recordando que
la suma de los enteros desde 1 hasta 𝑛 es 𝑛(𝑛 + 1)/2 es fácil obtener
𝑛(𝑛 + 1) 𝑛2 + 𝑛 + 2
1 + 1 + 2 + 3 +··· +𝑛 = 1 + = .
2 2
Hay otras generalizaciones y problemas similares a los cuales se puede aplicar el mismo
método.

v. Método de Singapur
El método Singapur es mundialmente famoso por tratarse de una forma de enseñanza de la
matemática que ha dado extraordinarios resultados en aquel país, y que se aplica en cerca de
50 países. Este método se caracteriza por su concepción de la matemática y por ser un método
específico de enseñanza, que impone la manera de abordar los problemas matemáticos.

La matemática, en el método Singapur, se concibe inicialmente como una herramienta para


resolver problemas, en esta perspectiva las matemáticas son importantes como una
herramienta para resolver problemas, al servicio de las personas. La enseñanza de la
matemática en el método de Singapur se basa en dos elementos: la estructura de la enseñanza
y el currículum y se caracteriza por tres tópicos esenciales que le han permitido fomentar su
propia estrategia de resolución de problemas.

• La enseñanza da inicio por lo concreto, con la utilización de materiales y ejercicios


que permiten a los estudiantes familiarizarse con los conceptos matemáticos desde la
vida cotidiana. Es decir, se parte de un problema o situación de la vida real sin
introducir formalidades propias de la disciplina.

• Se fomenta la representación visual del problema. Así, en dibujos, gráficos, matrices


u otra forma de representación los estudiantes organizan los datos conocidos (y
desconocidos) que atañen al problema.

• El currículum en el método Singapur tiene una concepción en espiral. En ella existen


variadas oportunidades de aprender un contenido o desarrollar una habilidad, pero no
repitiendo lo mismo, sino teniendo reiteradas ocasiones de resolver problemas que
involucren contenidos o habilidades similares.

46
A continuación, se muestran dos ejemplos de resolución de problemas aplicando el Método
de Singapur.

1. Un número disminuido en sus 8/𝟏𝟏 resulta 18. ¿Cuánto se le restó al número?


Solución
Método de barra
En este método se utiliza una barra para representar de manera concreta,
8
Esta barra representa el número del que se van a restar los 11 , para que de 18

8 3
Como sabemos que después de restarle al número sobran 18 que es equivalente ,
11 11
entonces 18 entre 3 es igual a 6. Luego el número que se resta es 8(6) = 48 y el número dado
es 11(6) = 66, como se muestra en la gráfica de más abajo.

11
2. Si los 20 del volumen de un depósito está ocupado, por cierta sustancia química, para llenar
5
el depósito, se necesita $540 ¿Cuánto cuesta de litro de dicha sustancia química,
3
sabiendo que el depósito tiene una capacidad de 400 litros.

Solución
11
Como los del volumen del depósito está ocupado y el recipiente tiene capacidad de 400
20
litros, cada parte del recipiente es 20 litros.

En la segunda tabla se representa la parte que está llena y como se sabe que la parte vacía se
llena con 540 pesos y como son 9 las partes vacías se divide 540 entre 9 y cada 20 litros
cuestan 60 pesos.

47
Luego para resolver el problema se aplica una regla de tres como se muestra mas
20 litros ∶ $60 5
{ 5 } ⇒ x(20) = 3 (60) ⇒ x = 5
∶ x
3
5
Así que 3 de litros de la sustancia química cuesta 5 pesos

Ejercicios 4.1
Resolver los siguientes problemas:

Problema 1
Si 𝑆(𝑛) denota la suma de los dígitos de un número natural n, encuentre todas las soluciones
de 𝑛 (𝑆(𝑛) − 1) = 2010 y muestre que son las únicas.

Problema 2
Sea 𝑃(𝑛) = (𝑛2 − 𝑛 + 1)(𝑛2 + 3𝑛 + 1). Halle todos los enteros 𝑛 para los cuales 𝑃(𝑛)
es un entero positivo primo.

Problema 3
Sea 𝑋 un conjunto arbitrario de 20 números naturales. Probar que hay al menos dos elementos
de 𝑋 cuya diferencia es un múltiplo de 19.

Problema 4
Supongamos que en una reunión hay 𝑛 personas y nos preguntamos por el número de personas
que conoce cada una. Convenimos que, si una persona conoce a otra, ésta también conoce a
la primera; y que “nadie se conoce a sí mismo”. Probar que hay al menos dos personas que
tienen el mismo número de conocidos.

Problema 5
Sea 𝑆 un conjunto de números naturales. Mostrar que hay un subconjunto 𝑇 ∈ 𝑆 tal que la suma
de todos sus elementos es un múltiplo de |𝑠|.

Problema 6
Hallar la suma de los primeros 2015 enteros positivos y múltiplos de 2015 positivos.

Problema 7
Se tienen los números del 1 al 2016 escritos en una pizarra. Se tachan 1007 de ellos. Probar
que entre los números que quedaron sin tachar en la pizarra, hay al menos 2 de ellos que son
consecutivos.

48
Problema 8
En las ventanas del apartamento de Luisa se van a escribir 12 números dígitos no nulos de
manera que las sumas de los números escritos en cada fila sean iguales y las sumas de los
números escritos en cada columna sean iguales, ya se han escrito 7 números, termine de
colocar los números

Problema 9
Un número se denomina interesante si es un natural menor que 21 y su cuadrado sumado con
el cuadrado de un número primo es 21. Halle todos los números interesantes.

Problema 10
Pedro tiene 2, 015 cajas. Cada caja contiene 2, 015 cubos. Cada cubo, de una caja, contiene
una cantidad distinta de puntos y ninguno tiene más de 2, 015 puntos. Además, cada caja tiene
pintado una cantidad distinta de puntos no mayor que 2, 015 ¿Cuántos puntos tienen los
objetos de Pedro?

Problema 11
Un número es de Joel si es un natural menor que 2015 y su cuadrado sumado con el cuadrado
de un número cuadrado perfecto es 2015. Halle la suma de todos los números de Joel.

Problema 12
(2𝑥 2 +4𝑥+18)(7−𝑥)
Hallar todos los enteros positivos 𝑥 tales que la expresión represente un
−3𝑥 2 +18𝑥+21
número entero.

Problema 13
𝑛(𝑛+1)
Demostrar que la suma de los 𝑛 primeros enteros positivos para 𝑛 ≥ 1 es .
2

Problema 14
Margarita construyó un cuadrado de lado 1, un cuadrado de lado 2, un cuadrado de lado 3 y
así continuó agregando 1 al lado del cuadrado anterior hasta construir un cuadrado de lado
2015 ¿Cuál es el área máxima que puede cubrir con estos cuadrados?

49
Problema 15
Un número natural seguido de 2015 ceros se denomina condenado, así, el primer número
condenado es 1(10)2015 , el segundo es 2(10)2015 , el tercer número condenado es 3(10)2015,
y así sucesivamente. Determine el residuo de dividir el 18134 número condenado por 9.

Problema 16
¿Cuál es el residuo de dividir 201537 − 2015 por 383838?

Problema 17
Se dispone de pequeñas piezas de madera de tamaño 4(5)(10). Decidir si es posible o no
apilarlas, sin dejar huecos y apoyándolas siempre sobre cualquiera de sus caras, para formar
un ortoedro de dimensiones ( 22015 )(32015 )(52015 ).

Problema 18
Demostrar que para cualquier número natural 𝑥, se tiene que el número 𝑥 2 + 𝑥 es un número
par.

Problema 19 (OME -2007)


Determinar todos los posibles valores enteros no negativos que puede tomar la expresión
𝑚2 +𝑚𝑛+𝑛2
siendo 𝑚 y 𝑛 enteros no negativos tales que 𝑚𝑛 ≠ 1.
𝑚𝑛−1

Problema 20 (OME -2007)


¿Cuáles son los números enteros positivos que se pueden obtener de exactamente 2007
maneras distintas, como la suma de al menos dos números enteros positivos consecutivos?
¿Cuál es el menor de todos ellos?. Ejemplo: el número 9 se escribe exactamente de dos
maneras distintas: 9 = 4 + 5,
9 = 2 + 3 + 4

Problema 21 (OME 2006)


Probar que el producto de cuatro naturales consecutivos no puede ser ni cuadrado ni cubo
perfecto.

Problema 22 (OME 2005)


Sean 𝑎 y 𝑏 enteros. Probar que la ecuación
(𝑥 − 𝑎)(𝑥 − 𝑏)(𝑥 − 3) + 1 = 0 admite a lo sumo una solución entera.

Problema 23 (OME-2004)
¿Existe alguna potencia de 2 que al escribirla en el sistema decimal tenga todos sus dígitos
distintos de cero y sea posible reordenar los mismos para formar con ellos otra potencia de 2?
Justificar la respuesta

50
Problema 24 (OME-2002)
Sea 𝑛 un número natural y 𝑚 el que resulta al escribir en orden inverso las cifras de 𝑛.
Determinar, si existen, los números de tres cifras que cumplen 2𝑚 + 𝑆 = 𝑛, siendo 𝑆 la
suma de las cifras de 𝑛.

Problema 25
Sea 𝑛 un número entero positivo llamemos 𝐴(𝑛) el número de divisores positivos de 𝑛
incluyendo a 1 y a 𝑛 y sea
𝑆(𝑛) = 𝐴(1) + 𝐴(2) + ⋯ + 𝐴(𝑛). Sea 𝑎 el número de enteros positivos 𝑛 ≤ 2015 con 𝑆(𝑛)
impar ) y sea 𝑏 el número de enteros positivos 𝑛 ≤ 2015 con 𝑆(𝑛) par. Hallar |𝑎 − 𝑏|

Problema 26 (OMCC, Honduras, 2008).


Halle el menor entero positivo 𝑁 tal que la suma de sus cifras sea 100, y la suma de las cifras de
2𝑁 sea 110.

Problema 27
Un número entero es de Peniel si puede expresarse como
𝑃 = 96𝑘+1 + 32𝑘+1 ∗ 4872𝑘 − 12 para algún entero positivo 𝑘. Demuestre que todo número de
Peniel es múltiplo de 730.

PROBLEMA 28
Un número entero positivo 𝐾 se denomina de Karolyn si la suma de sus cifras sea 2015, y la
suma de las cifras de 2𝐾 sea 2023. Hallar el menor número de Karolyn.

Problema 29 (OMCC 2001)


Encontrar todos los números naturales 𝑁 que cumplan las dos condiciones siguientes:
➢ Sólo dos de los dígitos de 𝑁 son distintos de 0 y uno de ellos es 3.
➢ N es un cuadrado perfecto.

Problema 30 (IMO 2002)


Los divisores positivos del entero 𝑛 > 1 son
𝑑1 < 𝑑2 < ⋯ < 𝑑𝑘 con 𝑑1 = 1 y 𝑑𝑘 = 𝑛 . Sea
𝑑 = 𝑑1 𝑑2 + 𝑑2 𝑑3 + ⋯ + 𝑑𝑘−1 𝑑𝑘 . Pruebe que 𝑑 < 𝑛2 y halle todos los 𝑛 para los cuales 𝑑 divide
a 𝑛2 .

Problema 31 (IMO 2006-4)


Determine todas las parejas de enteros (𝑥, 𝑦) tales que
1 + 2𝑥 + 22𝑥+1 + 1 = 𝑦 2 .

51
Problema 32 (IMO 2003-6)
Sea 𝑝 un número primo. Demostrar que existe un número primo 𝑞 tal que, para todo entero 𝑛, el
número 𝑛𝑝 − 𝑝 no es divisible por 𝑞.

Problema 33
La media aritmética de 20 números es 30 y la media aritmética de otros 30 números es 20. ¿Cuál
es la media aritmética de los 50 números?

Problema 34

Encuentre todos los enteros positivos 𝑎, 𝑏, 𝑐 tales que 𝑎𝑏 + 𝑏𝑐 + 𝑐𝑎 es número primo y


𝑎+𝑏 𝑏+𝑐
=
𝑎+𝑐 𝑏+𝑎
Problema 35
𝑥+𝑦
Demostrar que si 𝑥 e 𝑦 son reales no negativos entonces ≥ √𝑥𝑦.
2

Problema 36

Si se sabe que la ecuación 𝑥 3 + 𝑚𝑥 2 + 𝑥 + 𝑛 = 0 tiene raíces reales positivas cuyos cuadrados


suman 1, ¿cuánto valen 𝑚, 𝑛 y las raíces?

Problema 36 (IMO 1964).

Sean 𝑎, 𝑏 y 𝑐 los lados de un triángulo. Pruebe que


𝑎2 (−𝑎 + 𝑏 + 𝑐) + 𝑏 2 (𝑎 − 𝑏 + 𝑐) + 𝑐 2 (𝑎 + 𝑏 − 𝑐) ≤ 3𝑎𝑏𝑐

VI. Criterio de Evaluación: se tomarán en cuenta los siguientes criterios:


a) La participación en los trabajos: individual, en pequeño grupo y gran grupo.
b) La calidad y creatividad en los trabajos individuales.
c) La entrega de los trabajos en el plazo establecido y la organización de los mismos.

Puntuación:
Trabajos prácticos asignados para realizarlos tanto individual como en grupo: ……...40%
Exámenes de reconocimiento: uno de medio término (30%) y otro final (30%)………60%

Práctica Ejercicios Fecha de entrega Puntuación


1 1.1; 1.2 ; 1.3 Viernes 20/04/2018 10
2 1.4; 2.1 Viernes 27/04/2018 10
3 2.2; 3.1 Viernes 04/05/2018 10
4 3.2; 4.1 Viernes 11/05/2018 10

Viernes 27/04/2018 Viernes 11/05/2018


Examen de medio término (30%) Examen final (30%)

52
VII. Bibliografía Y Referencias
Centro Nacional de Evaluación para la Educación Superior, A.C. (Ceneval) (2013). Guía
del examen nacional de ingreso a la Educación Superior (EXANI-II) (10ma octava ed.).
Recuperado de http://www.uabc.mx/eventos/ceneval/GuiadelEXANI-II2013.pdf

Fernández, M., Preisseer, A., Segura, L, Falcon, Yolanda (1996). Lógica elemental.
Recuperado de as/logica/FERNANDEZ_DE_CASTRO_MAX_Logica_Elemental.pdf

Fridman, L. M. (1995). Metodología para resolver problemas de Matemática. (m. en C.J.


Ramón Jiménez, Traducción). México, D. F.: Grupo editorial Iberoamericana.

Gómez, J.P. R. & Molina, A. (2000). Potenciar la Capacidad de aprender y pensar (2da
ed. Rev.). Madrid, España.: Narcea, S. A.

Miller, C. D., Heeren, V.E. & Hornsby, E. J. (1999). Matemática: Razonamiento y


Aplicaciones. (8va, ed.) Naucalpan de Juárez, México. Addison Wesley Longman.

Pascal, L.C. Matemática Moderna. Madrid, España. Editorial Pirámide.

Rodríguez, B., Cruz del C. & González, S. (2008). El proceso de razonamiento lógico y
la enseñanza Actualidad contable FACES. Año 11 No. 17, julio-diciembre. Mérida, Venezuela.
Pág. 111-121.
Acevedo González, Georffrey. Lógica Matemática. Medellín, Colombia, 2011

53
ANEXO

54

S-ar putea să vă placă și